Peds Exam 1

Pataasin ang iyong marka sa homework at exams ngayon gamit ang Quizwiz!

What is the primary purpose of prescribing a histamine receptor antagonist for an infant diagnosed with gastroesophageal reflux? a. Prevent reflux b. Increase gastric acid production. c. Prevent hematemesis. d. Reduce gastric acid production.

d. Reduce gastric acid production.

Why does the rate of respiratory infections increase from 3-6mos of age? a) the exposure to pathogens is decreased b) maternal antibodies have decreased c) the diameter of the airway is larger in the infant than that of an older child d) viral agents that are milder in older children are more severe in infants

b) maternal antibodies have decreased

What intervention should the nurse implement when a 5 year old tells the nurse, "I need a Band-Aid" after having an injection. a. Ask why he/she wants a Band-Aid. b. Apply a Band-Aid. c. Explain why a Band-Aid is not needed. d. Show he/her that the bleeding has already stopped.

b. Apply a Band-Aid.

A young child with human immunodeficiency virus is receiving several antiretroviral drugs. What is the expected outcome of these drug therapies? Select one: a. Prevent spread of disease b. Cure the disease c. Delay disease progression d. Treat Pneumocystis jiroveci pneumonia

c. Delay disease progression

Nurses must be alert for increased fluid requirements when a child presents with which possible concern? a. Mechanical ventilation b. Increased intracranial pressure (ICP) c. Fever d. Congestive heart failure

c. Fever

Which strategy would be the least appropriate for a child to use to cope? a. Learning problem solving b. Using relaxation techniques c. Having parents solve problems d. Listening to music

c. Having parents solve problems

Which statement correctly describes β-thalassemia major (Cooley's anemia)? Select one: a. All formed elements of the blood are depressed. b. Inadequate numbers of red blood cells are present. c. Increased incidence occurs in families of Mediterranean extraction. d. Increased incidence occurs in persons of West African descent.

c. Increased incidence occurs in families of Mediterranean extraction.

What is an advantage of peritoneal dialysis? Select one: a. Dietary limitations are not necessary. b. Treatments are done in hospitals. c. Parents and older children can perform treatments. d. Protein loss is less extensive.

c. Parents and older children can perform treatments.

What is the single most important factor to consider when communicating with children? Select one: a. The child's physical condition b. The presence or absence of the child's parent c. The child's developmental level d. The child's nonverbal behaviors

c. The child's developmental level

A nurse is admitting a 4-year-old child to the pediatric unit and making plans to help the child adjust to the unit. The nurse understands that a 4-year-old child's greatest fear related to hospitalization is a fear of what? 1 Bodily harm 2 Lack of control 3 Loss of independence 4 Separation from mother

1 Bodily harm The psychosexual development of a preschooler is focused on the fear of invasive procedures.

What pain scale is used to measure the intensity of pain in preschoolers? 1 FACES scale 2 Visual analogue scale 3 Numerical rating scale 4 Verbal descriptor scale

1 FACES scale

A toddler has frequent temper tantrums. The parents ask a nurse how to limit this acting-out behavior. What should the nurse recommend? 1 Ignore the tantrum whenever possible. 2 Restrain the child whenever a tantrum begins. 3 Move the child to a quiet area as soon as a tantrum begins. 4 Visit the clinic to request medication to control the tantrum.

1 Ignore the tantrum whenever possible. Ignoring the temper tantrum as long as the child is not causing self-harm avoids reinforcement of the behavior.

Which stage of Erikson's theory would the nurse explain describes the development of a preschooler? 1 Initiative versus guilt 2 Trust versus mistrust 3 Identity versus role confusion 4 Autonomy versus sense of shame and doubt

1 Initiative versus guilt According to Erikson's theory, the initiative versus guilt stage is applicable to preschoolers between the ages of 3 to 6 years.

A nurse is performing a respiratory assessment of an 8-month-old child with the diagnosis of viral pneumonia. The nurse identifies bronchial breath sounds over areas of consolidation, mild substernal retractions, profuse mucus production, pallor, and a temperature of 102° F (38.9° C). What is the priority nursing action? 1 Suctioning the nasopharynx so a patent airway can be maintained 2 Starting an intravenous infusion to provide necessary fluids and electrolytes 3 Calling the respiratory therapist to start preparations for oxygen administration 4 Notifying the practitioner of the fever so a prescription for an antipyretic can be issued

1 Suctioning the nasopharynx so a patent airway can be maintained

The nurse is providing care to a preschool-age client of Asian descent during a scheduled health maintenance visit. The family speaks fluent English. Which assessment strategies should the nurse implement with the child and family based on the current data? Select all that apply. 1 Using open-ended questions 2 Avoiding prolonged eye contact 3 Phrasing questions in a neutral manner 4 Asking all questions directly to the interpreter 5 Asking several questions for time management purposes

1 Using open-ended questions 2 Avoiding prolonged eye contact 3 Phrasing questions in a neutral manner

A prescription for an isotonic enema is written for a 2-year-old child. What is the maximal amount of fluid the nurse should administer without a specific prescription from the healthcare provider? 1 100 to 150 mL 2 155 to 250 mL 3 255 to 360 mL 4 365 to 500 mL

3 255 to 360 mL

When does the anterior fontanel of an infant close? 1 At 4 to 10 months 2 At 8 to 12 months 3 At 12 to 18 months 4 At 18 to 26 months

3 At 12 to 18 months

Which heart sound is normally heard in a toddler that is considered abnormal in an adult over 30-years-old? 1 S1 2 S2 3 S3 4 S4

3 S3

What does the nurse state is the cause of frequent upper respiratory tract infections in toddlers? 1 Stress 2 Unhealthy diet 3 Lack of exercise 4 Immature immune system

4 Immature immune system

The nurse encourages the mother of a toddler with acute laryngotracheobronchitis (LTB) who is in the ER to stay at the bedside as much as possible. The nurse's rationale for this action is primarily that: A. Mothers of hospitalized toddlers often experience guilt. B. The mother's presence will reduce anxiety and ease child's respiratory efforts. C. Separation from mother is a major developmental threat at this age. D. The mother can provide constant observations of the child's respiratory efforts.

B. The mother's presence will reduce anxiety and ease child's respiratory efforts.

Which description of a stool is characteristic of intussusception? a. Loose, foul-smelling stools b. "Currant jelly" stools c. Hard stools positive for guaiac d. Ribbon-like stools

b. "Currant jelly" stools

The nurse is talking to the parent of a 13-month-old child. The mother states, "My child does not make noises like 'da' or 'na' like my sister's baby, who is only 9 months old." Which statement by the nurse would be most appropriate to make? a. "You should ask other parents what noises their children made at this age." b. "I am going to request a referral to a hearing specialist." c. "You should not compare your child to your sister's child." d. "I think your child is fine, but we will check again in 3 months."

b. "I am going to request a referral to a hearing specialist."

A child with autism spectrum disorder (ASD) is admitted to the hospital with pneumonia. The nurse should plan which priority intervention when caring for the child? Select one: a. Maintain frequent touch and eye contact with the child. b. Maintain a structured routine and keep stimulation to a minimum. c. Take the child frequently to the playroom to play with other children. d. Place the child in a room with a roommate of the same age.

b. Maintain a structured routine and keep stimulation to a minimum.

The nurse is assessing a child with acute epiglottitis. Examining the child's throat by using a tongue depressor might precipitate which symptom or condition? Select one: a. Respiratory tract infection b. Sore throat c. Complete obstruction d. Inspiratory stridor

c. Complete obstruction

A preschool child is being admitted to the hospital with dehydration and a urinary tract infection (UTI). Which urinalysis result should the nurse expect with these conditions? a. WBC >2; specific gravity 1.016 b. WBC <1; specific gravity 1.008 c. WBC >2; specific gravity 1.030 d. WBC <2; specific gravity 1.025

c. WBC >2; specific gravity 1.030

The nurse inserts and IV in the arm of a 2-year-old. What would be an expected response from the child? a. Facial grimacing and decreased po intake b. Tries to be brave and not cry if that's what he/she believes the nurse expects it. c. Crying and trying to hit the nurse d. Begging the nurse to put the IV in at a later time

c. Crying and trying to hit the nurse

According to Erikson's theory of psychosocial development, what is the correct order of a child's behavior as they age? 1. The child concentrates on work and play. 2. The child develops autonomy by making choices. 3. The child is concerned about appearance and body image. 4. The child develops feelings of superego or conscience.

2, 4, 1, 3

A parent brings a 2-month-old infant with Down syndrome to the pediatric clinic for a physical and administration of immunizations. Which clinical finding should prompt the nurse to perform further assessment? 1 Flat occiput 2 Small, low-set ears 3 Circumoral cyanosis 4 Protruding furrowed tongue

3 Circumoral cyanosis

The nurse is providing care to an infant diagnosed with Down syndrome. Which parental statement related to the infant's growth indicates the need for further education? 1 "My baby will have growth deficiencies during infancy." 2 "My child will have accelerated growth during adolescence." 3 "My child will most likely be overweight by 3 years of age." 4 "My baby will have reduced growth in both height and weight."

2 "My child will have accelerated growth during adolescence."

A 3-week-old infant has surgery for esophageal atresia. What is the immediate postoperative nursing care priority for this infant? 1 Giving the oral feedings slowly 2 Reporting vomiting to the practitioner 3 Checking the patency of the nasogastric tube 4 Monitoring the child for signs of infection at the incision site

3 Checking the patency of the nasogastric tube

A nurse educates a mother about the proper administration of oral medication to her 4-year-old child. What statement made by the mother indicates effective learning? 1 "I should administer the medication with a cup or spoon." 2 "I should mix the medicine in a large amount of food." 3 "I should avoid giving a straw to my child to take pills." 4 "I should use a disposable oral syringe to prepare liquid doses."

4 "I should use a disposable oral syringe to prepare liquid doses."

In which stage of development are food choices affected by such factors as peer acceptability and sociability, possibly resulting in malnutrition or chronic illness? 1 Midlife 2 Infancy 3 Childhood 4 Adolescence

4 Adolescence

A nurse reviews with the parents of a young infant the principles of growth and development. Place the milestones in the order of their usual achievement. 1. Waves bye-bye and sits alone 2. Climbs stairs and drinks from a cup 3. Draws a vertical line and walks on tiptoe 4. Walks alone and builds a tower of two blocks 5. Sits momentarily without support and rolls over

5, 1, 4, 2, 3

Which clinical manifestation would most suggest acute appendicitis? a. Abdominal pain that is most intense at McBurney point b. Abdominal pain that is relieved by eating c. Bright red or dark red rectal bleeding d. Rebound tenderness

a. Abdominal pain that is most intense at McBurney point

A newborn assessment shows separated sagittal suture, oblique palpebral fissures, depressed nasal bridge, protruding tongue, and transverse palmar creases. These findings are most suggestive of: Select one: a. Down syndrome. b. cerebral palsy. c. microcephaly. d. fragile X syndrome.

a. Down syndrome.

What is the most common cause of hearing impairment in children? Select one: a. Congenital ear defects b. Chronic otitis media c. Auditory nerve damage d. Congenital rubella

b. Chronic otitis media

An infant diagnosed with pyloric stenosis experiences excessive vomiting that can result in which condition? a. Hyperchloremia b. Hypernatremia c. Metabolic alkalosis d. Metabolic acidosis

c. Metabolic alkalosis

What is the primary result of anemia? Select one: a. Presence of abnormal hemoglobin. b. Depressed hematopoietic system. c. Increased blood viscosity. d. Decreased oxygen-carrying capacity of blood.

d. Decreased oxygen-carrying capacity of blood.

Which foods should the nurse include when teaching a group of school-age clients regarding appropriate nutritional intake? Select all that apply. 1 Bananas 2 Fried chicken 3 Low-fat yogurt 4 Whole-wheat dinner rolls 5 Sugary, carbonated beverages

1 Bananas 3 Low-fat yogurt 4 Whole-wheat dinner rolls

A severely dehydrated infant with gastroenteritis is admitted to the pediatric unit. Nothing-by-mouth (NPO) status is prescribed. The parents ask why their baby cannot be fed. The nurse explains that it is necessary to do what? 1 Correct electrolyte imbalances 2 Allow the intestinal tract to rest 3 Determine the cause of the diarrhea 4 Prevent perianal irritation from the diarrhea

2 Allow the intestinal tract to rest

A nurse in the pediatric clinic is assessing an 8-year-old child who has had asthma since infancy. What clinical finding requires immediate intervention? 1 Barrel chest 2 Audible wheezing 3 Heart rate of 105 beats/min 4 Respiratory rate of 30 breaths/min

2 Audible wheezing

What is the most common cause of death among adolescents? 1 Suicide 2 Homicide 3 Accidents 4 Substance abuse

3 Accidents

A nurse is assessing a toddler with vesicoureteral reflux. What clinical finding does the nurse expect to identify? 1 Dysuria 2 Oliguria 3 Glycosuria 4 Proteinuria

1 Dysuria Discomfort during urination (dysuria) is a symptom of a urinary tract infection (UTI), which is common with vesicoureteral reflux. During voiding, urine is swept up the ureters and then flows back to the bladder, resulting in a residual volume that provides a medium for the development of a UTI.

An adolescent girl is concerned about her body image after amputation of a leg for bone cancer. After the nurse has obtained the girl's consent, what nursing action is most therapeutic? 1 Encouraging her peers to visit 2 Keeping her lower body covered 3 Placing her in a room by herself 4 Limiting her visitors to the family

1 Encouraging her peers to visit

Which pain scale should a nurse use to measure the intensity of pain in toddlers? 1 FACES scale 2 Visual analogue scale 3 Numerical rating scale 4 Verbal descriptor scale

1 FACES scale

What in students is being assessed when a school nurse conducts audiometric screenings? 1 Hearing acuity 2 Sensorineural hearing loss 3 Auditory processing deficits 4 Hearing problems caused by wax

1 Hearing acuity

A nurse is reviewing the laboratory report of an adolescent child with nephrotic syndrome. What does the nurse expect analysis of the child's urine to reveal? 1 High protein level 2 Low specific gravity 3 Numerous red blood cells 4 Several crystalline particles

1 High protein level

The parents of a 14-month-old boy with bilateral cryptorchidism ask the nurse in the pediatric clinic why it is important for him to have surgery before he is 2 years old. Before responding, the nurse takes into consideration the fact that uncorrected cryptorchidism can result in what? 1 Infertility 2 Hydrocele 3 Varicocele 4 Epididymitis

1 Infertility

A 3-year-old child is hospitalized with nephrotic syndrome. The child has oliguria and generalized edema. What factor does the nurse identify that will have the greatest effect on the child's adjustment to hospitalization? 1 Lack of parental visits 2 Inability to select a variety of foods 3 Response of peers to the edematous appearance 4 Willingness to participate in cooperative play activities

1 Lack of parental visits

An infant with a diagnosis of failure to thrive has been receiving enteral feedings for 3 days. All feedings have been retained, but the skin and mucous membranes are dry, and the infant has lost weight. What should the nurse do first in light of these findings? 1 Notify the practitioner 2 Document the assessment findings 3 Increase the fluid component in the feeding 4 Increase the calorie component of the feeding

1 Notify the practitioner Dry mucous membranes and weight loss are classic signs of dehydration. The nurse should calculate the infant's fluid requirements, then obtain a prescription from the practitioner to increase either free water or the amount of the feedings as needed.

A child undergoes tonsillectomy and adenoidectomy for numerous recurrent respiratory tract infections. After the surgery, what should the nurse teach the parents to do? 1 Offer crushed ice chips. 2 Encourage the intake of ice cream. 3 Keep the child in the supine position. 4 Gargle with a diluted mouthwash solution.

1 Offer crushed ice chips. Ice chips are soothing and promote vasoconstriction.

A nurse plans to talk to the parents of a toddler about toilet training. What should the nurse tell the parents is the most important factor in the process of toilet training? 1 Parents' attitude about it 2 Child's desire to remain dry 3 Child's ability to sit still on the toilet 4 Parents' willingness to work at the toilet training

1 Parents' attitude about it The parents' attitude, approach, and understanding of the child's physical and psychologic readiness are essential to letting the child proceed at his or her own pace with appropriate parental intervention.

A 3-year-old boy in respiratory distress is treated in the emergency department. A diagnosis of acute spasmodic laryngitis (spasmodic croup) is made. At the time of discharge, the mother asks how to handle another attack at home. What should the nurse recommend? 1 Placing him near a cool-mist humidifier 2 Bringing him to the emergency department 3 Giving him an over-the-counter cough syrup 4 Offering him warm tea sweetened with honey

1 Placing him near a cool-mist humidifier During a spasmodic croup attack, cool humidified air to decrease inflammation is a fast home remedy.

Which finding in the urinalysis of a 5½-year-old child will alert the nurse to consider the possibility of lead-induced kidney damage? 1 Protein 2 Calcium 3 Potassium 4 Phosphate

1 Protein Protein is usually not excreted in urine because it is a large molecule. When found, it indicates kidney disease.

What should a nurse include in the plan of care for a 9-year-old child with nephrotic syndrome? 1 Providing meticulous skin care 2 Restricting fluids to 4 oz (120 mL) each shift 3 Offering a diet low in carbohydrates and protein 4 Sending blood to the laboratory for typing and crossmatching

1 Providing meticulous skin care

A 2-year-old child is brought to the emergency department after the sudden onset of high fever, drooling, and respiratory distress. What nursing actions should the nurse perform? Select all that apply. 1 Start an intravenous line. 2 Draw a blood sample for a complete blood count and differential. 3 Examine the child's throat with a flashlight and tongue depressor for swelling. 4 Assess oxygen saturation of the blood and administer oxygen by mask if it is below 94%. 5 Ask the parents to remain in the waiting room during the examination and interventions. 6 Assess the child's temperature and administer an antipyretic if the rectal temperature is higher than 101° F (38.3° C).

1 Start an intravenous line. 2 Draw a blood sample for a complete blood count and differential. 4 Assess oxygen saturation of the blood and administer oxygen by mask if it is below 94%. 6 Assess the child's temperature and administer an antipyretic if the rectal temperature is higher than 101° F (38.3° C). The child is presenting with signs of epiglottitis. An intravenous line will prevent dehydration and provide access for emergency medications if needed. Oxygen saturation should be kept above 94% to prevent hypoxia. Determination of the cause of the epiglottis (bacterial versus viral) will be necessary to manage the illness. Children in this age range can have febrile seizures; antipyretics will control the temperature.

A 6-year-old child with acute spasmodic bronchitis who is receiving humidified air removes the mask, and while bathing the child the nurse notes increasing respiratory distress. What is the most appropriate nursing intervention? 1 Stopping the bath and replacing the mask 2 Performing postural drainage and clapping the chest 3 Placing the child in the orthopneic position and calling the practitioner 4 Suctioning the child's nasal passages and waiting for the dyspnea to subside

1 Stopping the bath and replacing the mask

The mother of a 5-year-old girl child reports to a nurse that her daughter has a genital discharge and recurrent urinary tract infections. What should the nurse suspect from the mother's statement? 1 The child may be a victim of sexual abuse. 2 The child may be a victim of physical abuse. 3 The child may be a victim of physical neglect. 4 The child may be a victim of emotional neglect.

1 The child may be a victim of sexual abuse.

At 18 months of age a child born with a cleft lip and palate is readmitted for palate surgery. Why does the nurse teach the parents not to brush their child's teeth immediately after the surgery? 1 The suture line might be injured. 2 A toothbrush might be frightening. 3 The child will probably have no teeth. 4 A toothbrush has not been used before.

1 The suture line might be injured.

Which toddler behaviors should the nurse identify as ritualism during the health history portion of the assessment? Select all that apply. 1 Using the same cup with each meal 2 Being able to use a spoon efficiently 3 Eating the same foods as other family members 4 Refusing to eat if the different foods are touching 5 Rejecting a meal because it is served in a different bowl

1 Using the same cup with each meal 4 Refusing to eat if the different foods are touching 5 Rejecting a meal because it is served in a different bowl

What should a nurse incorporate into the plan of care for a school-aged child hospitalized with acute glomerulonephritis (AGN)? Select all that apply. 1 Weighing daily 2 Restricting fluids 3 Monitoring intravenous therapy 4 Instituting isolation precautions 5 Checking the blood pressure hourly

1 Weighing daily 2 Restricting fluids Comparing daily weights is an objective measure of fluid balance and response to diuretic therapy. Fluids, as well as sodium, are restricted in the presence of oliguria. Intravenous therapy is not needed unless there is an emergency. Isolation is unnecessary because the illness is not communicable. Although the blood pressure is closely monitored, it need not be taken hourly.

A nurse in the pediatric intensive care unit is assessing a 6-month-old infant with bronchiolitis. What physiologic responses to this lower respiratory tract infection does the nurse expect? Select all that apply. 1 Wheezing 2 Bradycardia 3 Sternal retractions 4 Nasal flaring 5 Prolonged expiratory phase

1 Wheezing 3 Sternal retractions 4 Nasal flaring 5 Prolonged expiratory phase Bronchiolitis in most infants is caused by respiratory syncytial virus. Wheezing occurs as the air passages narrow, resulting in the typical whistling sound. As breathing becomes more difficult, the infant must expend more energy and use accessory muscles of respiration to breathe. Nasal flaring is a predominant characteristic of bronchiolitis. The infectious and inflammatory changes narrow the bronchial passage, making it difficult for air to leave the lungs. As a result of increased respiratory effort and decreased oxygen exchange, tachycardia, not bradycardia, develops. Breath sounds are diminished because of edema of the bronchiolar mucosa and filling of the lumina with mucus and exudate.

Name three differences between pediatric and adult respiratory systems that put children at great risk with a respiratory illness.

1) Everything is smaller- easier for infection to spread 2) Everything is narrower- any swelling at all causes significant decreased surface area for breathing 3) Ability to take a deep breath when they are SOB is decreased because they don't have as developed of chest muscles- rib cage is more flexible and doesn't allow them to take a deep breath 4) Immune system function isn't fully functional- antibodies from mother drops off at 3months

Name three factors that put an infant or child with bronchiolitis at risk for hospital admission?

1) Someone who is more symptomatic (tachycardia, tachypneic) 2)) Underlying health issues, esp cardiac or respiratory 3) Dehydrated 4) Age- the younger they are esp if less than 6mos or if they were born premature

Which parental statement regarding the use of car safety seats for a toddler-age child indicates the need for further education by the nurse? 1 "I should follow the manufacturer instructions regarding use." 2 "I should allow my child to ride unrestrained for short car trips." 3 "I should allow my child to have a toy for quiet play while in the car." 4 "I should encourage my child to help attach the buckles."

2 "I should allow my child to ride unrestrained for short car trips."

The nurse is providing nutritional guidance to the parents of a preschool-age client. Which parental comment would prompt the nurse to provide further education? 1 "We allow our child to drink only pasteurized apple cider." 2 "We let our child sample cookie dough while making cookies." 3 "We always wash our hands well before any food preparation." 4 "We use separate utensils for food preparation and for eating."

2 "We let our child sample cookie dough while making cookies."

A nurse is teaching parents about the nutritional needs of their 15-year-old child. What information should the nurse provide? Select all that apply. 1 Increase the child's fat intake 2 Provide iron-rich foods to the child 3 Increase the child's daily protein intake 4 Curb the child's diet to help prevent obesity 5 Provide adequate vitamin supplementation to your child

2 Provide iron-rich foods to the child 3 Increase the child's daily protein intake The nurse should instruct the parents to provide an iron-rich diet to the child. This will help prevent anemia. The nurse should instruct the parents to increase the child's protein intake. Vitamin and mineral supplements are not required, so the nurse should instruct the parents to avoid them.

A 6-week-old infant has just been found to have gastroesophageal reflux. What teaching is most important to discuss with the parents at this time? 1 Feeding cereal with a spoon 2 Providing formula thickened with cereal 3 Placing the infant on its back immediately after feedings 4 Explaining changes in care after surgical repair of the esophageal defect

2 Providing formula thickened with cereal

The nurse educates an obese adolescent about healthy dietary habits and risk associated with obesity. Which statement by the adolescent indicates the need for further counseling? 1 "I should do exercise." 2 "I should play more outdoor games." 3 "I should watch more TV to reduce the stress." 4 "I should modify my diet and have lots of vegetables and water."

3 "I should watch more TV to reduce the stress."

The mother of an 18-month-old child with a cleft palate asks the nurse why the pediatrician has recommended that closure of the palate be performed before the child is 2 years old. How should the nurse respond? 1 "As the child gets older, the palate gets wider and more difficult to repair." 2 "Eruption of the 2-year molars often complicates the surgical procedure." 3 "You need to have the surgery performed before your child starts to use faulty speech patterns." 4 "After a child is 2 years old, surgery is frightening, so you need to avoid it if at all possible."

3 "You need to have the surgery performed before your child starts to use faulty speech patterns."

A 1-week-old infant has been in the pediatric unit for 18 hours after placement of a spica cast. The nurse obtains a respiratory rate slower than 24 breaths/min; no other changes are noted. Because the infant is apparently well, the nurse does not report or document the slow respiratory rate. Several hours later the infant experiences severe respiratory distress, and emergency care is necessary. What should be considered if legal action is taken? 1 Most infants' respirations are slow when they are uncomfortable. 2 The respirations of young infants are irregular, so a drop in rate is unimportant. 3 Vital signs that are outside the expected parameters are significant and should be documented. 4 The respiratory tracts of young infants are underdeveloped, and the respiratory rate is not significant.

3 Vital signs that are outside the expected parameters are significant and should be documented.

A 7-year-old child must remain quietly in bed while undergoing peritoneal dialysis. What activity is most appropriate for the nurse to plan for this child? 1 Learning to play chess 2 Constructing a model airplane 3 Working multiple-piece puzzles with another child 4 Using a large sponge ball to play catch with a roommate

3 Working multiple-piece puzzles with another child Working puzzles is a quiet activity that will not jeopardize placement of the peritoneal catheter and is appropriate for the child's cognitive level and allows social interaction with a peer. Chess requires cognitive abilities beyond the scope of a 7-year-old child. Although constructing a model airplane is a quiet activity, it is probably too difficult for a 7-year-old to do without help from an adult.

According to Freud's theory, in what order do the stages of child development occur? 1. Anal 2. Latency 3. Oral 4. Genital 5. Oedipal

3, 1, 5, 2, 4 Freud's theory sets forth five stages of child development. The oral stage (stage 1) spans the period from birth to 12 to 18 months. The anal stage (stage 2) occurs between the ages of 18 months and 3 years. The Oedipal stage (stage 3) occurs between 3 and 6 years of age. The latency stage (stage 4) occurs between the ages of 6 and 12 years. Finally, the genital stage (stage 5) lasts from puberty through adulthood.

A nurse is making room assignments on the pediatric unit. Who is the best choice of roommate for a 10-year-old boy with juvenile idiopathic arthritis? 1 An 11-year-old girl with colitis 2 A 10-year-old boy with asthma 3 A 10-year-old girl with a fractured femur 4 An 11-year-old boy who has undergone splenectomy

4 An 11-year-old boy who has undergone splenectomy

A 30-month-old boy with cystic fibrosis is admitted to the pediatric unit with a severe upper respiratory infection. The toddler is small for his age. What pathologic process does the nurse know is the cause of his small stature? 1 Increased salt retention 2 An atrioventricular defect 3 Retention of carbon dioxide 4 An absence of pancreatic enzymes

4 An absence of pancreatic enzymes Fats, proteins, and carbohydrates are not digested because of a deficiency of pancreatic enzymes and therefore physical growth is hampered.

A teacher's aide in a kindergarten class informs the school nurse that a male student said that his mother beat him and that he has bruises on the back and shoulders. What is the priority nursing action? 1 Notifying Child Protective Services 2 Reporting this information to the principal 3 Calling the parents to arrange a conference 4 Assessing the child for the presence of bruises

4 Assessing the child for the presence of bruises

A toddler who lacks toilet training is admitted to a hospital. What does the nurse need to do when collecting urine samples from the toddler? Select all that apply. 1 Squeeze urine from the diaper. 2 Place a hat under the toilet seat. 3 Convince the child to void in the unfamiliar receptacle. 4 Attach single-use bags over the child's urethral meatus. 5 Use the terms for urination that the child can understand.

4 Attach single-use bags over the child's urethral meatus. 5 Use the terms for urination that the child can understand.

The parents of a preschooler tell the nurse that they try to inculcate good eating habits by asking the child to be at the table until the "plate is clean." What condition is the child at risk for? 1 Anorexia 2 Depression 3 Aggression 4 Poor eating habits

4 Poor eating habits

A nurse working the 7 am to 3 pm shift is caring for a 14-year-old adolescent for whom intake and output are being monitored. The primary healthcare provider prescribes an intravenous infusion to be administered at a rate of 50 mL/hr. The adolescent had 4 oz (120 mL) of milk and a muffin for breakfast at 8:30 am. At 9 am the adolescent vomited 200 mL. At 10 am the adolescent had 60 mL of water with medications. At 11 am the adolescent voided 550 mL of urine. For lunch, at 12:30 pm, the adolescent ate 3 oz (90 mL) of soup and 4 oz (120 mL) of ice cream. The adolescent voided 450 mL at 2 pm. Calculate the adolescent's total intake for the 7 am to 3 pm shift. Record your answer using a whole number. _____ mL

790 mL

What is the characteristic of the preoperational stage of cognitive development? a. Generalizations can be made. b. Thinking is concrete. c. Thinking is logical. d. Reasoning is inductive.

???

What intervention is appropriate for a child with bronchiolitis? A. Place the infant upright with the neck flexed B. Ensure that the assigned RN does not care for other high-risk children C. Administered pancreatic supplements with meals and snacks. D. Place the child on airborne precautions.

B. Ensure that the assigned RN does not care for other high-risk children Bronciolitis is highly contagious so you don't want the RN caring for a child with this disease and also a high risk patient. A is wrong because with the neck flexed it would be harder to breathe.

A child is admitted to the hospital with the diagnosis of laryngotracheobronchitis (LTB). The nurse should be prepared to: A. administer antibiotics and assist with possible intubation. B. administer nebulized epinephrine and oral or IM dexamethasone. C. swab the throat for a throat culture. D. obtain a sputum specimen.

B. administer nebulized epinephrine and oral or IM dexamethasone. laryngotracheobronchitis is a form of croup. Typically croup is VIRAL so would not give ATBs. Swabbing for throat culture can cause irritation. Sputum specimen would only be if we suspected bacterial. Racemic epinephrine = nebulized ephinephrine Dexamethasone = steroid; want systemic steroids to rapidly decrease inflammation

How long should a child with strep throat be on isolation?

For 24 hrs after starting ATBs. They can return to school when they feel well enough to once those 24hrs of ATBs are up.

β-Adrenergic agonists, such as albuterol (Ventolin), are often prescribed for a child with an asthma attack for what resulting action? a. Dilation of the bronchioles b. Liquefaction of secretions c. Reduction of existing infection d. Reduction of inflammation of the lungs

a. Dilation of the bronchioles

The nurse, caring for a neonate with a suspected tracheoesophageal fistula, should include what intervention into the plan of care? a. Elevating the head to facilitate secrete drainage. b. Elevating the head for feedings only. c. Feeding glucose water only. d. Avoiding suctioning unless the infant is cyanotic.

a. Elevating the head to facilitate secrete drainage.

Chelation therapy is begun on a child with β-thalassemia major with what expected result? Select one: a. Elimination of excess iron. b. Managing nausea and vomiting. c. Treatment of the disease. d. Decreasing the risk of hypoxia.

a. Elimination of excess iron.

Which statement expresses accurately the genetic implications of cystic fibrosis (CF)? a. If it is present in a child, both parents are carriers of this defective gene. b. It is inherited as an autosomal dominant trait. c. It is a genetic defect found primarily in non-Caucasian population groups. d. There is a 50% chance that siblings of an affected child also will be affected.

a. If it is present in a child, both parents are carriers of this defective gene.

A parent whose child has been diagnosed with a cognitive deficit should be counseled about what fact related to intellectual impairment? Select one: a. Is likely caused by a variety of factors. b. Is usually caused by parental intellectual impairment. c. Is usually due to a genetic defect. d. Is rarely due to first-trimester events.

a. Is likely caused by a variety of factors.

Which pain assessment tool is most appropriate for assessing an adolescent? Select one or more: a. Numeric Scale b. Wong-Bake Faces Scale c. NIPS d. rFLACC

a. Numeric Scale

A nurse is preparing to complete an admission assessment on a 2-year-old child who is sitting on the parent's lap. Which technique should the nurse implement to complete the physical examination? a. Perform the examination while the child is on the parent's lap. b. Ask the child to stand by the parent while completing the examination. c. Ask the parent to place the child in the hospital crib. d. Take the child and parent to the examination room.

a. Perform the examination while the child is on the parent's lap.

What is the most common mode of transmission of human immunodeficiency virus (HIV) in the pediatric population? Select one: a. Perinatal transmission b. Poor hand washing c. Sexual abuse d. Blood transfusions

a. Perinatal transmission

Which function of play is a major component of play at all ages? a. Sensorimotor activity b. Socialization c. Creativity d. Intellectual development

a. Sensorimotor activity

What represents the major stressor of hospitalization for children from middle infancy throughout the preschool years? a. Separation anxiety b. Fear of pain c. Loss of control d. Fear of bodily injury

a. Separation anxiety

Which factor predisposes a child to urinary tract infections? a. Short urethra in young girls b. Frequent emptying of the bladder c. Increased fluid intake d. Prostatic secretions in males

a. Short urethra in young girls

A parent asks the nurse why a developmental assessment is being conducted for a child during a routine well-child visit. The nurse answers based on what knowledge about such routine developmental assessments? Select one: a. The best method for early detection of cognitive disorders. b. Not necessary unless the parents request them. c. Valuable in measuring intelligence in children. d. Frightening to parents and children and should be avoided.

a. The best method for early detection of cognitive disorders.

When a child diagnosed with chronic renal failure, the progressive deterioration produces a variety of clinical and biochemical disturbances that eventually are manifested in the clinical syndrome known as what? Select one: a. Uremia b. Proteinuria c. Pyelonephritis d. Oliguria

a. Uremia

Which pain assessment tool is most appropriate for a 6-year-old Select one or more: a. Wong-Baker Faces Scale b. rFLACC c. NIPS d. Numeric Scale

a. Wong-Baker Faces Scale

Which statement by a parent about a child's conjunctivitis indicates that further teaching is needed? a. "I'll notify my doctor if the eye gets redder or the drainage increases." b. "When the eye drainage improves, we'll stop giving the antibiotic ointment." c. "After taking the antibiotic for 24 hours, my child can return to school." d. "I'll have separate towels and washcloths for each family member."

b. "When the eye drainage improves, we'll stop giving the antibiotic ointment."

The predominant characteristic of the intellectual development of the child ages 2 to 7 years is egocentricity. What best describes this concept? a. Selfishness b. Inability to put self in another's place c. Self-centeredness d. Preferring to play alone

b. Inability to put self in another's place

Which pain assessment tool should be used with a neonate? a. Wong-Baker Faces Scale b. NIPS c. Numeric Scale d. Visual analog scale

b. NIPS

Which of the following are complimentary therapy methods that can be used to treat pain in an infant? (select all that apply) Select one or more: a. Magic glove b. Touch/massage c. Patterned breathing d. Sucrose e. Kangaroo care

b. Touch/massage d. Sucrose e. Kangaroo care

According to Erikson, by the time children reach their 12 birthday, they should have learned to trust others and should have developed a sense of: a. intimacy. b. industry. c. integrity. d. identity.

b. industry.

What is a common side effect of corticosteroid therapy? a. Fever b. Hypertension c. Increased appetite d. Weight loss

c. Increased appetite

Which teaching guideline helps prevent eye injuries during sports and play activities? a. Restrict helmet use to those who wear eyeglasses or contact lenses. b. Discourage the use of goggles with helmets. c. Wear eye protection when participating in high risk sports such as paintball. d. Wear a face mask when playing any sport or playing roughly.

c. Wear eye protection when participating in high risk sports such as paintball.

What should the nurse recommend to prevent urinary tract infections in young girls? a. Cleansing the perineum with water after voiding b. Increasing fluids; decreasing salt intake c. Wearing cotton underpants d. Limiting bathing as much as possible

c. Wearing cotton underpants

A toddler with croup is in a cool mist tent. He is afraid and tries to climb out. What is the most appropriate action? a) tell the mother the child must stay in the tent b) place a toy in the tent to increase comfort c) administer a mild sedative d) let his mother hold him and direct the cool mist over his face

d) let his mother hold him and direct the cool mist over his face Being with the parent is something that is calming and comforting for a toddler. If we let them be with their safe space person, that helps control their fear and still allows for the cool mist to be applied.

Which of the following should be included in discharge instructions for a child with asthma? A) Keep home humidity above 50% B) use only feather pillows C) eliminate exposure to all cats and dogs d) take long-term medication even in the absence of symptoms

d) take long-term medication even in the absence of symptoms Tip: Be careful of questions that use concrete words like always, never, only, all, etc.

The nurse is conducting teaching for an adolescent being discharged to home after a renal transplantation. The adolescent needs further teaching if which statement is made? Select one: a. "I will observe my incision for any redness or swelling." b. "I won't miss doing kidney dialysis every week." c. "I will report any fever to my primary health care provider." d. "I am glad I only have to take the immunosuppressant medication for 2 weeks."

d. "I am glad I only have to take the immunosuppressant medication for 2 weeks."

A nurse is providing a parent information regarding autism spectrum disorder (ASD). Which statement made by the parent indicates understanding of the teaching? Select one: a. "Autism can be treated effectively with medication." b. "Childern with autism have hearing problems and prefer loud sounds." c. "Autism is characterized by periods of remission and exacerbation." d. "The onset of autism usually occurs before toddler stage."

d. "The onset of autism usually occurs before toddler stage."

In which cultural group is good health considered to be a balance between yin and yang? a. Australian aborigines b. African-Americans c. Native Americans d. Asians

d. Asians

What is the term used to identify when the meatal opening is located on the dorsal surface of the penis? Select one: a. Chordee b. Phimosis c. Hypospadias d. Epispadias

d. Epispadias

Which is the most common congenital anomaly associated with Down syndrome? a. Hypospadias b. Congenital hip dysplasia c. Pyloric stenosis d. Heart defects

d. Heart defects

The nurse is caring for a child diagnosed with an acute respiratory infection. What nursing intervention should be included in the plan of care? a. Asses respiratory rate once per shift b. Instruct patient to eat a high-protein diet c. Enocurage at least 4 ounces of fluid per hour d. Monitor pulse oximetry continously

d. Monitor pulse oximetry continously

The nurse is taking a sexual history on an adolescent girl. The best way to determine whether she is sexually active is to: Select one: a. ask both the girl and her parent if she is sexually active. b. ask her, "Are you sexually active?" c. ask her, "Are you having sex with a boyfriend?" d. ask her, "Are you having sex with anyone?"

d. ask her, "Are you having sex with anyone?"

The type of injury a child is especially susceptible to at a specific age is most closely related to: a. number of responsible adults in the home. b. educational level of the child. c. physical health of the child. d. developmental level of the child.

d. developmental level of the child.

An 8-year-old girl asks the nurse how the blood pressure apparatus works. The most appropriate nursing action is to: Select one: a. ask her why she wants to know. b. determine why she is so anxious. c. tell her she will see how it works as it is used. d. explain in simple terms how it works.

d. explain in simple terms how it works.

Question text The nurse must assess a child's capillary refilling time. This can be accomplished by: Select one: a. inspecting the chest. b. auscultating the heart. c. palpating the apical pulse. d. palpating the skin to produce a slight blanching

d. palpating the skin to produce a slight blanching

What name is given to identify an acquired hemorrhagic disorder that is characterized by excessive destruction of platelets? Select one: a. Immune thrombocytopenia b. Thalassemia major c. Disseminated intravascular coagulation d. Aplastic anemia

i got this wrong :(

What clinical finding does a nurse expect when assessing a 4-month-old infant with gastroenteritis and moderate dehydration? 1 Urine output of 50 mL/hr 2 Depressed anterior fontanel 3 History of allergies to certain formulas 4 Capillary refill time of less than 2 seconds

2 Depressed anterior fontanel

The nurse plans to perform an abdominal assessment of a 10-year-old child with suspected appendicitis. List in order of priority the techniques the nurse should use when assessing this child's abdomen. 1. Auscultating for bowel sounds 2. Asking where it hurts 3. Assessing the abdomen by touch 4. Visually examining the abdomen 5. Warming the stethoscope's diaphragm

2, 4, 5, 1, 3

The parents of a 2-year-old child are watching the nurse administer the Denver II Developmental Screening Test to their child. They ask, "Why did you make our child draw on paper? We don't let our child draw at home." What is the best response by the nurse? 1 "I should have asked you about drawing first." 2 "These drawings help us determine your child's intelligence." 3 "It lets us test the child's ability to perform tasks requiring the hands." 4 "I don't understand why drawing is forbidden in your home."

3 "It lets us test the child's ability to perform tasks requiring the hands."

An infant with a diaphragmatic hernia undergoes corrective surgery. What nursing assessment indicates that the infant's respiratory condition has improved? 1 Cessation of crying 2 Retention of 1 oz (30 mL) of formula 3 Reduction of arterial blood pH to 7.31 4 Auscultation of breath sounds bilaterally

4 Auscultation of breath sounds bilaterally

A nurse is evaluating a 3-year-old child's developmental progress. The inability to perform which task indicates to the nurse that there is a developmental delay? 1 Copying a square 2 Hopping on one foot 3 Catching a ball reliably 4 Using a spoon effectively

4 Using a spoon effectively

A 3-year-old child, who has been diagnosed with asthma, is being prescribed albuterol (Ventolin) for acute episodes. The parents should be advised that the child may exhibit which of the following common side effects of the medication? A. Insomnia B. Lethargy C. Constipation D. Weight gain

A. Insomnia Other side effects include tachycardia, agitation, and excitability.

What are indications that a child with acute epiglottis may be experiencing an airway obstruction? A) nasal flaring and bradycardia B) leaning forward with the chin thrust out C) leaning back and supporting self on hands and arms D) sore throat

B) leaning forward with the chin thrust out This is describing the tripod position.

A 17-year-old high school student with a history of asthma is brought to the emergency department. The nurse performs a physical assessment and identifies that the adolescent is experiencing an acute asthma exacerbation. Identify the assessments that support this conclusion. (Select all that apply.) A. Temperature 99.6 oF oral B. Dry, non-productive cough C. Wheezing on expiration D. Oxygen saturation 91% on room air E. Respiratory rate 40

B. Dry, non-productive cough C. Wheezing on expiration D. Oxygen saturation 91% on room air E. Respiratory rate 40 17 year old should have normal adult RR of 12-18. Medications that can be anticipated are a duoneb and steroids Duoneb = albuterol and Atrovent

You are caring for a child with a pulse oximeter. The low O2 sat alarm goes off. What should you do first? A. Count the child's respiratory rate. B. Look for circumoral cyanosis. C. Observe the child's level of consciousness. D. See if the photodector is placed correctly.

C. Observe the child's level of consciousness. If the pulse ox alarm is sounding, your first task is to physically look at the patient and see if there is a logical explanation for why their O2 sats are dropping.

A nurse is charting that a hospitalized child has labored breathing. Which medical term describes labored breathing? a. Tachypnea b. Hypopnea c. Dyspnea d. Orthopnea

c. Dyspnea

The urinary output of a 9-year-old child with acute glomerulonephritis decreases to 250 mL/24 hr. A diet low in sodium and potassium is prescribed. What should the nurse encourage the child to have for lunch? 1 Baked chicken, green beans, and lemonade 2 Cream of tomato soup, salami sandwich, and cola 3 Grilled cheese sandwich, sliced tomatoes, and milk 4 Peanut butter and jelly sandwich, celery, and orangeade

1 Baked chicken, green beans, and lemonade The foods in this grouping have the least sodium and potassium.

An infant has been admitted with failure to thrive (FTT). The nurse knows that more education is needed when one of the parents makes what statement? 1 "I can double the amount of water in the formula to save money." 2 "I need to hold her head up a little higher than her stomach when I feed her." 3 "I need to burp the baby when the feeding is done to get rid of swallowed air." 4 "I need to make sure that the formula is in the nipple so she doesn't swallow so much air."

1 "I can double the amount of water in the formula to save money."

A nurse is educating a group of mothers about the nutritional needs of toddlers. Which of these statements made by a mother indicate the need for further teaching? Select all that apply. 1 "I need to give more than 24 ounces of milk per day." 2 "I should provide more calories from fats than proteins." 3 "I should refrain from giving grapes, nuts, and raw vegetables." 4 "I should give foods rich in calcium and phosphorus for healthy bone growth." 5 "I should try to give small, frequent meals consisting of breakfast, lunch, and dinner."

1 "I need to give more than 24 ounces of milk per day." 2 "I should provide more calories from fats than proteins." Toddlers who consume more than 24 ounces of milk daily in place of other foods may develop milk anemia, because milk is a poor source of iron. A toddler needs more protein than fats in the diet.

Which questions should the nurse include when conducting a health history interview with the parents of a 4-year-old client to assess fine motor skills? Select all that apply. 1 "Is your child able to use scissors?" 2 "Is your child able to ride a tricycle?" 3 "Is your child able to tie shoe laces?" 4 "Is your child able to climb stairs using alternate feet?" 5 "Is your child able to build a tower using 9 or 10 blocks?"

1 "Is your child able to use scissors?" 3 "Is your child able to tie shoe laces?"

How should a nurse respond to parents who are concerned about separation anxiety in their 15-month-old toddler? 1 "This is an expected developmental reaction." 2 "You may be spending too much time with your child." 3 "It might be helpful to leave your child with someone once in a while." 4 "Toddlers who have separation anxiety may have difficulty when they start school."

1 "This is an expected developmental reaction." Understanding that separation anxiety is an expected developmental occurrence will be reassuring to the parents

After her baby undergoes corrective surgery for hypertrophic pyloric stenosis, the mother is asked to offer the first feeding. The infant sucks it eagerly and vomits immediately. What is the nurse's explanation to the mother? 1 "This often occurs after the first feeding." 2 "The baby is ridding postoperative mucus." 3 "Your feeding technique may need to be changed." 4 "Feedings will have to be stopped until peristalsis improves."

1 "This often occurs after the first feeding."

A nurse is reviewing previous education with the parents and a 12-year-old child with cystic fibrosis (CF). The disease was diagnosed when the child was 3 years old and has resulted in one hospitalization for a respiratory infection. For which potential complications of CF should the family be alert? Select all that apply. 1 Diabetes 2 Hematuria 3 Nasal polyps 4 Prolapsed rectum 5 Pulmonary infections 6 Urinary tract infections

1 Diabetes 3 Nasal polyps 4 Prolapsed rectum 5 Pulmonary infections By adolescence almost 50% of children with CF will have abnormal glucose tolerance test results as a result of the disease's effects on the pancreas

Which between-meal snack should a nurse tell the parents of a preschooler with a urinary tract infection to offer their child? 1 Skim milk 2 Fresh fruit 3 Hard candy 4 Cream soup

1 Skim milk A high-protein, high-carbohydrate snack provides additional nutrients to combat an infection and a fever. Also, fluid helps flush the urinary tract. Fruit does not provide the protein needed for the healing process. Candy provides empty calories. A cream soup is too heavy for a between-meal snack and does not provide the needed protein.

Which strategy needs to be employed while interviewing the adolescent as a part of her health-screening? 1 To start with more sensitive issues 2 To explain the limits of confidentiality 3 To ask more of close-ended questions 4 To interview the adolescent along with her parents

2 To explain the limits of confidentiality

How should a nurse assess a 4-year-old child with abdominal pain? 1 By asking the child to point to where it hurts 2 By auscultating the child's abdomen for bowel sounds 3 By observing position and behavior while the child is moving 4 By questioning the parents about their child's eating and bowel habits

3 By observing position and behavior while the child is moving

The nurse is providing anticipatory guidance to the parents of a child who has a difficult temperament. Which statement should the nurse include in the teaching session with these parents regarding their child's temperament? Select all that apply. 1 "Your child will develop predictable habits." 2 "Your child will benefit from a bedtime routine." 3 "Your child is open to change and adapts easily." 4 "Your child is likely to adapt slowly to the new daycare setting." 5 "Your child will display intense emotions, including those associated with happiness."

2 "Your child will benefit from a bedtime routine." 4 "Your child is likely to adapt slowly to the new daycare setting." 5 "Your child will display intense emotions, including those associated with happiness."

A 30-month-old toddler is brought to the emergency department in acute respiratory distress, and a diagnosis of laryngotracheobronchitis (viral croup) is made. What is the most important equipment for the nurse to have available when the child is admitted to the pediatric unit? 1 Intravenous set 2 Tracheotomy set 3 Nasal cannula for oxygen 4 Crib with padded side rails

2 Tracheotomy set A patent airway is the priority. A tracheotomy set should be kept immediately available in case of complete obstruction of the airway.

The nurse is providing care to a 6-week-old infant who is hospitalized for poor growth. The infant is currently being breastfed and is diagnosed with failure to thrive (FTT). Which is the priority nursing assessment for this infant? 1 Family financial difficulties 2 Uncoordinated suck and swallow 3 Neglect and abuse by the parents 4 Knowledge deficit related to nutritional intake

2 Uncoordinated suck and swallow Most cases of poor growth and FTT in the first two months of life occur due to an uncoordinated suck and swallow during feedings (formula or breast); therefore, this is the priority nursing assessment. Assessing for financial difficulties, neglect and abuse, and a knowledge deficit are appropriate but not the priority in this situation.

What is the correct order of the steps in which the nurse should administer eardrops to a toddler? 1. Place the toddler in the side-lying position. 2. Clean the outer ear using a washcloth. 3. Keep the toddler in the side-lying position for 2 to 3 minutes. 4. Pull the auricle down and back to straighten the ear canal. 5. Using a finger, apply gentle massage or pressure to the tragus. 6. Hold the dropper 1 cm above the ear canal to instill the prescribed drops.

2, 1, 4, 6, 3, 5

After several episodes of abdominal pain and vomiting, a 5-month-old infant is admitted with a tentative diagnosis of intussusception. What assessment should the nurse document that will aid confirmation of the diagnosis? 1 Frequency of crying 2 Amount of oral intake 3 Characteristics of stools 4 Absence of bowel sounds

3 Characteristics of stools Because intussusception creates intestinal obstruction in which the intestine "telescopes" and becomes trapped, passage of intestinal contents is lessened; stools are red and look like currant jelly because of the mixing of stool with blood and mucus.

A nurse provides instructions to a group of adolescents about ways to prevent obesity. Which statements made by an adolescent indicates a need for further learning? Select all that apply. 1 "I should avoid trans fats." 2 "I should limit portion sizes." 3 "I should consume a high-fat diet." 4 "I should take highly refined starch food." 5 "I should watch television for four hours only."

3 "I should consume a high-fat diet." 4 "I should take highly refined starch food." 5 "I should watch television for four hours only." A high-fat diet should be avoided by adolescents. Highly refined starches and sugars should be avoided because they are rich in calories. Adolescents should be advised to watch less than two hours of television per day. Most dieticians and nutrition experts recommend a diet with no trans fats. Adolescents should limit portion sizes to improve body weight.

Which teaching point regarding safety should the nurse include in the instructions for the parents of a school-age client who is a latchkey child? 1 "Consider getting a pet for your child." 2 "Plan play dates for your child to attend on afternoons you are not home." 3 "Teach your child not to display the keys used to enter the home after school." 4 "Provide structured activities for your child to complete while they are home alone."

3 "Teach your child not to display the keys used to enter the home after school."

Based on Erikson's theory of development, what should the nurse suggest to a mother of a toddler who harshly disciplines her child? 1 "Your behavior may induce a feeling of isolation in your child." 2 "You need to establish a sense of trust or the child may lose trust in you." 3 "Your child needs support and love or may develop feelings of shame and doubt." 4 "You need to control the child's impulses or the child may suffer guilt and frustration."

3 "Your child needs support and love or may develop feelings of shame and doubt." According to Erikson, a child of 2 years old is in the autonomy versus sense of shame and doubt stage. The nurse should counsel the mother that harshly disciplining her child may lead to a feeling of shame and doubt.

The nurse instructs the unlicensed assistive personnel (UAP) to obtain vital signs from four clients. From which client can the nurse instruct the UAP to obtain a radial pulse? 1 A 1-year-old child 2 An 18-month-old child 3 A 30-month-old child 4 A 6-month-old child

3 A 30-month-old child A satisfactory pulse can be taken radially in children older than 2 years of age, hence, the nurse can instruct the UAP to obtain a radial pulse from a 30-month-old child.

When picked up by a parent or the nurse, an 8-month-old infant screams and seems to be in pain. After observing this behavior, what should the nurse discuss with the parent? 1 Accidents and the importance of their prevention 2 Limiting playtime with other children in the family 3 Any other behaviors that the parent might have noticed 4 Food and specific vitamins that should be given to infants

3 Any other behaviors that the parent might have noticed

A child who recently returned from a three-day camping trip over spring vacation is brought to the clinic after a rash, chills, and low-grade fever develop. What are the most important data for the nurse to assess when taking the child's history? Select all that apply. 1 Date of return to school 2 Sports played on camping trip 3 Tendency to allergic reactions 4 Duration of signs and symptoms 5 Recent exposure to poison oak or ivy

3 Tendency to allergic reactions 4 Duration of signs and symptoms 5 Recent exposure to poison oak or ivy

The nurse is assessing a newborn and anticipates that the newborn has renal impairment. Which finding supports the nurse's conclusion? 1 The newborn has odorless urine. 2 The newborn has colorless urine. 3 The newborn first voids after 76 hours. 4 The newborn's urine has a specific gravity of 1.020.

3 The newborn first voids after 76 hours. A newborn should void within 24 hours. However, in this case, the newborn first voids after 76 hours, indicating renal impairment.

What is the best room assignment for a 5-year-old child admitted with injuries that may be related to abuse? 1 In an isolation room 2 With a friendly older child 3 With a child of the same age 4 In a room near the nurses' desk

4 In a room near the nurses' desk

When teaching a class about parenting, the nurse asks the participants what they do when their toddlers have a temper tantrum. Which statement demonstrates one father's understanding of the origin of temper tantrums? 1 After a temper tantrum he disciplines his child by restricting a favorite food or activity. 2 When a temper tantrum begins he isolates and ignores his child until the behavior improves. 3 During a temper tantrum he partially gives in to his child before the tantrum becomes excessive. 4 He tries to prevent a temper tantrum by allowing his child to choose between two reasonable alternatives.

4 He tries to prevent a temper tantrum by allowing his child to choose between two reasonable alternatives. This parent's action gives his child more control by allowing the child to make a decision. This demonstrates an understanding of what the toddler can and cannot do safely.

What teaching must the nurse emphasize to the family when preparing a school-aged child with persistent asthma for discharge? 1 A cold, dry environment is desirable. 2 Limits should not be placed on the child's behavior. 3 The health problem is gone when symptoms subside. 4 Medications must be continued even when the child is asymptomatic.

4 Medications must be continued even when the child is asymptomatic.

The nurse should assess an infant with gastroesophageal reflux for what complication? 1 Bowel obstruction 2 Abdominal distention 3 Increased hematocrit 4 Respiratory problems

4 Respiratory problems

Which of the following would most likely be contraindicated in the care of a child with CF? A) forced expiration B) supplemental oxygen as needed C) aerobic exercise D) percussion and postural drainage

B) supplemental oxygen as needed Oxygen shouldn't be delivered as desired. Don't give extra oxygen just because they want it because it will further promote decreased respiratory drive. In general, they should be participating in the other listed activities.

A 7-year-old child is brought to the Emergency Department for an acute asthma attack. He is wheezing, tachypneic, diaphoretic, and looks frightened. The nurse should prepare to administer: A. IV methylprednisolone (Solu-medrol) B. Albuterol (Proventil) C. Cromolyn sodium (Intal) D. Ipratropium bromide (Atrovent)

B. Albuterol (Proventil) If this question was SATA, you would also give A and D BUT B would always be first. Always give albuterol FIRST to help open the lungs and allow for the other medications to work faster.

A child is admitted to the hospital with pneumonia. The child's oximetry reading upon admission to the pediatric floor is 88. The priority nursing activity for this child would be to: A. begin administration of intravenous fluids. B. begin oxygen per nasal cannula at 1 liter. C. obtain a blood sample to send to the lab for electrolyte analysis. D. medicate for pain.

B. begin oxygen per nasal cannula at 1 liter. Remember your ABCs when answering priority questions! 1 L is good start for peds patients as long as their O2 sat isn't too low

A nurse is assessing a preterm infant. The assessment that might indicate the infant's respiratory status is worsening is: A. an arterial CO2 of 40. B. grunting respirations with nasal flaring. C. acrocyanosis. D. periorbital edema.

B. grunting respirations with nasal flaring.

Which of the following represent early subtle signs of hypoxia? A) peripheral cyanosis B) hypotension C) central cyanosis D) change in level of consciousness

D) change in level of consciousness LOC is always one of the first indicators of hypoxia.

A 12-year-old with cystic fibrosis is to receive three Pancrease capsules five times a day. The nurse is aware that this medication is given: A. Promote excretion of fats B. Promote adequate oxygenation C. Prevent iron-deficiency anemia D. Facilitate utilization of nutrients

D. Facilitate utilization of nutrients Pancrease (krion) brings pancreatic enzymes so food can be broken down to absorb it and get the nutrients Goal is to get the nutrients not excrete

A child is admitted with epiglottitis, and needs an x-ray. The nurse orders the x-ray to be done by: A. portable x-ray in the unit's waiting room. B. transporting the child to the x-ray department by a cart. C. transporting the child to the x-ray department in a parent's arms. D. portable x-ray in the child's hospital room.

D. portable x-ray in the child's hospital room. Epiglottitis is a form of croup.

The leading cause of death from unintentional injuries in children is: a. poisoning. b. motor vehicle related fatalities. c. drowning. d. fire- and burn-related fatalities.

c. drowning.

Why does feeding position affect the risk of developing an ear infection in infants?

Short eustachian tubes makes it easier for fluid to collect in ears if they're laying down and creates a good medium for bacteria growth. You want to feed them when they are sitting upright.

What is a common side effect of corticosteroid therapy? Select one: a. Increased appetite b. Fever c. Weight loss d. Hypertension

a. Increased appetite

Which statement is most descriptive of pediatric family-centered care? a. It recognizes that the family is the constant in a child's life. b. It reduces the effect of cultural diversity on the family. c. It avoids expecting families to be part of the decision-making process. d. It encourages family dependence on the health care system.

a. It recognizes that the family is the constant in a child's life.

When caring for a child with suspected epiglottis, the nurse should: a) have intubation equipment available b) visually inspect the child's oropharynx with a tongue blade c) obtain a throat culture d) prepare to administer immunizations

a) have intubation equipment available

An 18-month-old child is seen in the clinic is diagnosed with acute otitis media (AOM). Oral amoxicillin is prescribed. Which statement made by the parent indicates a correct understanding of the instructions? a. "I should administer all the prescribed medication." b. "I will immediately stop giving medication if I notice a change in hearing." c. "I will stop giving medication if fever is still present in 24 hours." d. "I should continue medication until the symptoms subside."

a. "I should administer all the prescribed medication."

A mother brings 6-month-old Eric to the clinic for a well-baby checkup. She comments, "I want to go back to work, but I don't want Eric to suffer because I'll have less time with him." The nurse's most appropriate answer is: a. "Let's talk about the child care options that will be best for Eric." b. "You will need to stay home until Eric starts school." c. "I'm sure he'll be fine if you get a good baby-sitter." d. "You should go back to work so Eric will get used to being with others."

a. "Let's talk about the child care options that will be best for Eric."

The nurse is teaching the parent about the diet of a child experiencing severe edema associated with acute glomerulonephritis. Which information should the nurse include in the teaching? a. "You will need to avoid adding salt to your child's food." b. "You will need to decrease the number of calories in your child's diet." c. "Your child's diet will need an increased amount of protein." d. "Your child's diet will consist of low-fat, low-carbohydrate foods."

a. "You will need to avoid adding salt to your child's food."

When it is generally recommended that a child being treated for acute streptococcal pharyngitis may return to school? a. After taking antibiotics for 24 hours b. If no complications develop c. When the sore throat is better d. After taking antibiotics for 3 days

a. After taking antibiotics for 24 hours

An infant is brought to the emergency department with poor skin turgor, sunken fontanel, lethargy, and tachycardia. This is suggestive of which condition? a. Dehydration b. Overhydration c. Calcium excess d. Sodium excess

a. Dehydration

Which statement best explains why iron deficiency anemia is common during toddlerhood? Select one: a. Milk is a poor source of iron. b. Fetal iron stores are depleted by age 1 month. c. Iron cannot be stored during fetal development. d. Dietary iron cannot be started until age 12 months.

a. Milk is a poor source of iron.

Which of the following are recommended nursing interventions when caring for a child who is receiving opioid medications? (select all that apply) a. Monitor for respiratory rate and oxygen saturation b. Administer loperamide to control diarrhea c. Ensure that nalaxone (Narcan) is readily available d. Combine with use of non-pharmacologic and non-opioid methods to achieve pain control e. Administer by oral or intravenous routes rather than intramuscular routes

a. Monitor for respiratory rate and oxygen saturation c. Ensure that nalaxone (Narcan) is readily available d. Combine with use of non-pharmacologic and non-opioid methods to achieve pain control e. Administer by oral or intravenous routes rather than intramuscular routes

A Chinese toddler has pneumonia. The nurse notices that the parent consistently feeds the child only the broth that comes on the clear liquid tray. Food items such as Jell-O, Popsicles, and juices are left. What would best explain this? a. The parent is trying to restore normal balance through appropriate "hot" remedies. b. Hispanics believe that an innate energy called chi is strengthened by eating soup. c. The parent is trying to feed child only what child likes most. d. Hispanics believe that the "evil eye" enters when a person gets cold.

a. The parent is trying to restore normal balance through appropriate "hot" remedies.

When a child is diagnosed with chronic renal failure, the progressive deterioration produces a variety of clinical and biochemical disturbances that eventually are manifested in the clinical syndrome known as what? a. Uremia b. Proteinuria c. Oliguria d. Pyelonephritis

a. Uremia

Which of the following are recommended nursing actions for treating pain in a child? (select all that apply) Select one or more: a. Use round the clock scheduling if continued pain is anticipated b. Give IM injections of pain medicines before trying PO and IV routes c. Withhold opioids to reduce the risk of addiction d. Use an appropriate assessment tool based on the child's developmental level e. Choose a treatment method based on the child, pain history, and the situation

a. Use round the clock scheduling if continued pain is anticipated d. Use an appropriate assessment tool based on the child's developmental level e. Choose a treatment method based on the child, pain history, and the situation

What is an important consideration for the nurse who is communicating with a very young child? a. Use transition objects such as a doll. b. Disguise own feelings, attitudes, and anxiety. c. Speak loudly, clearly, and directly. d. Initiate contact with the child when the parent is not present.

a. Use transition objects such as a doll.

Which vitamin supplements are necessary for children with cystic fibrosis? Select one: a. Vitamins A, D, E, and K b. Vitamins B6 and B12 c. Magnesium d. Vitamin C and calcium

a. Vitamins A, D, E, and K

Which intervention is appropriate when examining a male infant for cryptorchidism? a. Warming the room b. Cooling the examiner's hands c. Taking a rectal temperature d. Eliciting the cremasteric reflex

a. Warming the room

As related to inherited disorders, which statement is descriptive of most cases of hemophilia? Select one: a. X-linked recessive inherited disorder in which a blood-clotting factor is deficient b. Y-linked recessive inherited disorder in which the red blood cells become moon shaped c. Autosomal dominant disorder causing deficiency in a factor involved in the blood-clotting reaction d. X-linked recessive inherited disorder causing deficiency of platelets and prolonged bleeding

a. X-linked recessive inherited disorder in which a blood-clotting factor is deficient

Which of the following are consequences of unrelieved pain in children? (Select all that apply) Select one or more: a. Pancreatitis b. Decreased pain threshold c. Increased blood glucose d. Altered sleep patterns e. Decreased oxygen saturation

b. Decreased pain threshold c. Increased blood glucose d. Altered sleep patterns e. Decreased oxygen saturation

Latasha, age 8 years, is being admitted to the hospital from the emergency department with an injury from falling off her bicycle. What intervention will help an 8 year old most in adjusting to a hospital admission? a. Orient her parents, because she is young, to her room and hospital facility. b. Explain hospital schedules such as mealtimes. c. Use terms such as "honey" and "dear" to show a caring attitude. d. Explain when parents can visit and why siblings cannot come to see her.

b. Explain hospital schedules such as mealtimes.

Which statement most accurately describes the pathologic changes of sickle cell anemia? Select one: a. Sickle-shaped cells carry excess oxygen b. Increased red blood cell destruction occurs c. Sickle-shaped cells decrease blood viscosity d. Decreased red blood cell destruction occurs

b. Increased red blood cell destruction occurs

The nurse is seeing an adolescent boy and his parents in the clinic for the first time. What should the nurse do first? a. Make the family comfortable. b. Introduce himself or herself. c. Explain the purpose of the interview. d. Give an assurance of privacy.

b. Introduce himself or herself.

Why do infants and young children quickly have respiratory distress in acute and chronic alterations of the respiratory system? a. The gag reflex increases mucus production. b. Mucus and edema obstruct small airways. c. They have a widened, shorter airway. d. There is a defect in their sucking ability.

b. Mucus and edema obstruct small airways.

Therapeutic management of the child with acute diarrhea and dehydration usually begins with what intervention? a. Antidiarrheal medications such as paregoric b. Oral rehydration solution (ORS) c. Adsorbents such as kaolin and pectin d. Clear liquids

b. Oral rehydration solution (ORS)

When a preschool child is hospitalized without adequate preparation, what is the child may likely see hospitalization as? a. Threat to child's self-image b. Punishment c. An opportunity for regression d. Loss of companionship with friends

b. Punishment

The nurse is planning activity for a 4-year-old child with anemia. Which activity should the nurse plan for this child? Select one: a. Game of "hide and seek" in the children's outdoor play area b. Puppet play in the child's room c. Participation in dance activities in the playroom d. A walk down to the hospital lobby

b. Puppet play in the child's room

Which age-group is most concerned with body integrity? a. Toddler b. School-age child c. Adolescent d. Preschooler

b. School-age child

Because of their striving for independence and productivity, which age-group of children is particularly vulnerable to events that may lessen their feeling of control and power? a. Preschoolers b. School-age children c. Infants d. Toddlers

b. School-age children

The nurse is observing parents playing with their 10-month-old daughter. What should the nurse recognize as evidence that the child is developing object permanence? a. She recognizes that a ball of clay is the same when flattened out. b. She looks for the toy the parents hide under the blanket. c. She returns the blocks to the same spot on the table. d. She bangs two cubes held in her hands.

b. She looks for the toy the parents hide under the blanket.

Cystic fibrosis (CF) is suspected in a toddler. Which test is essential in establishing this diagnosis? a. Bronchoscopy b. Sweat chloride test c. Urine creatinine d. Serum calcium

b. Sweat chloride test

What is the major focus of the therapeutic management for a child with lactose intolerance? a. Administration of daily normal saline enemas b. Teaching dietary modifications c. Compliance with the medication regimen d. Providing emotional support to family members

b. Teaching dietary modifications

What should the nurse keep in mind when planning to communicate with a child who is diagnosed with an autism spectrum disorder (ASD)? Select one: a. The child is expected to use sign language. b. The child may exhibit monotone speech and echolalia. c. The child is not listening if he/she is not looking at the nurse. d. The child has normal verbal communication.

b. The child may exhibit monotone speech and echolalia.

Which "expected outcome" would be developmentally appropriate for a hospitalized 4-year-old child? a. The child will be dressed and fed by the parents. b. The child will independently ask for play materials or other personal needs. c. The child will be able to verbalize an understanding of the reason for the hospitalization. d. The child will have a parent stay in the room at all times.

b. The child will independently ask for play materials or other personal needs.

A 4-month-old infant diagnosed with gastroesophageal reflux disease (GERD) is thriving without other complications. What should the nurse suggest to minimize reflux? a. Place in Trendelenburg position after eating. b. Thicken formula with rice cereal. c. Give continuous nasogastric tube feedings. d. Give larger, less frequent feedings.

b. Thicken formula with rice cereal.

A 9 year old diagnosed with Down syndrome is mainstreamed into a regular third-grade class for part of the school day. His mother asks the school nurse about programs such as Cub Scouts that he might join. The nurse's recommendation should be based on what knowledge? a. Programs such as Cub Scouts are inappropriate for children who are cognitively impaired. b. Parents of children with Down syndrome encourage programs such as scouting because they deny that their children have disabilities. c. Children with Down syndrome have the same need for socialization as other children. d. Children with Down syndrome socialize better with children who have similar disabilities.

c. Children with Down syndrome have the same need for socialization as other children.

The major cause of death for children older than 1 year is: a. congenital abnormalities. b. unintentional injuries. c. infection. d. cancer.

b. unintentional injuries.

Which of the following signs is an EARLY indicator of post op bleeding after tonsillectomy? a) decreased blood pressure b) restlessness c) frequent swallowing d) all of the above

c) frequent swallowing

What is a common cause of acute diarrhea? a. Hypothyroidism b. Hirschsprung's disease c. Antibiotic therapy d. Meconium ileus

c. Antibiotic therapy

The nurse is taking a health history on an adolescent. Which best describes how the chief complaint should be determined? a. Ask for a detailed listing of symptoms. b. Use what the adolescent says to determine, in correct medical terminology, what the problem is. c. Ask the adolescent, "Why did you come here today?" d. Interview the parent away from the adolescent to determine the chief complaint.

c. Ask the adolescent, "Why did you come here today?"

What is the appropriate way to use the rFLACC tool to assess pain? Select one or more: a. Vital signs must be assessed to score pain with this tool b. Use this tool on a sleeping child and the Numeric Scale when the child is awake c. Assess an awake child for 1 to 5 minutes or longer to determine rFLACC scores d. A pain intervention is not needed with tool for a score less than 6

c. Assess an awake child for 1 to 5 minutes or longer to determine rFLACC scores

Instructions for decongestant nose drops should include what recommendation? a. Keeping drops to use again for nasal congestion. b. Administering drops after feedings and at bedtime. c. Avoiding use for more than 3 days. d. Administering drops until nasal congestion subsides.

c. Avoiding use for more than 3 days.

The nurse observes some children in the playroom. Which play situation exhibits the characteristics of parallel play? a. Danielle playing with a music box on her mother's lap. b. Kimberly and Amanda sharing clay to each make things. c. Brian playing with his truck next to Kristina playing with her truck. d. Adam playing a board game with Kyle, Steven, and Erich.

c. Brian playing with his truck next to Kristina playing with her truck.

The nurse, caring for a child with acute renal failure, should recognize event as a sign of hyperkalemia? Select one: a. Dyspnea b. Seizure c. Cardiac arrhythmia d. Oliguria

c. Cardiac arrhythmia

The nurse, caring for a child with acute renal failure, should recognize which event as a sign of hyperkalemia? a. Seizure b. Oliguria c. Cardiac arrhythmia d. Dyspnea

c. Cardiac arrhythmia

The nurse is assessing a child with acute epiglottitis. Examining the child's throat by using a tongue depressor might precipitate which symptom or condition? a. Respiratory tract infection b. Inspiratory stridor c. Complete obstruction d. Sore throat

c. Complete obstruction

Which statement best describes why children have fewer respiratory tract infections as they grow older? Select one: a. Secondary infections rarely occur after viral illnesses. b. The amount of lymphoid tissue decreases. c. Repeated exposure to organisms causes increased immunity. d. Viral organisms are less prevalent in the population.

c. Repeated exposure to organisms causes increased immunity.

What term is used to identify the condition in which the normal adult hemoglobin is partly or completely replaced by abnormal hemoglobin? Select one: a. Aplastic anemia b. Iron deficiency anemia c. Sickle cell anemia d. Thalassemia major

c. Sickle cell anemia

When caring for a child with probable appendicitis, the nurse should be alert to recognize what sign of perforation? a. Decreased abdominal distention b. Bradycardia c. Sudden relief from pain d. Anorexia

c. Sudden relief from pain

What does the term equianalgesia refer to for a patient receiving morphine sulfate? a. The dosage of fetanyl to achieve the same effect as a dose of morphine sulfate. b. The maximum dose of morphine sulfate that can be given to achieve the desired effect. c. The amount of morphine sulfate needed to get the same effect with different routes of administration d. The combination of a non-opioid drug given along with morphine sulfate.

c. The amount of morphine sulfate needed to get the same effect with different routes of administration

Which immunization should be given with caution to children infected with human immunodeficiency virus? Select one: a. Pneumococcus b. Inactivated poliovirus c. Varicella d. Influenza

c. Varicella (live attinuated)

A preschool child is being admitted to the hospital with dehydration and a urinary tract infection (UTI). Which urinalysis result should the nurse expect with these conditions? Select one: a. WBC <2; specific gravity 1.025 b. WBC <1; specific gravity 1.008 c. WBC >2; specific gravity 1.030 d. WBC >2; specific gravity 1.016

c. WBC >2; specific gravity 1.030

During a funduscopic examination of a school-age child, the nurse notes a brilliant, uniform red reflex in both eyes. The nurse should recognize that this is: a. a sign of a possible visual defect; the child needs vision screening. b. a sign of small hemorrhages, which usually resolve spontaneously. c. a normal finding. d. an abnormal finding; the child needs referral to an ophthalmologist.

c. a normal finding.

The head-to-tail direction of growth is referred to as: a. sequential. b. proximodistal. c. cephalocaudal. d. mass to specific.

c. cephalocaudal.

The nurse is teaching the parent about the diet of a child experiencing severe edema associated with acute glomerulonephritis. Which information should the nurse include in the teaching? Select one: a. "Your child's diet will need an increased amount of protein." b. "Your child's diet will consist of low-fat, low-carbohydrate foods." c. "You will need to decrease the number of calories in your child's diet." d. "You will need to avoid adding salt to your child's food."

d. "You will need to avoid adding salt to your child's food."

Binocularity, the ability to fixate on one visual field with both eyes simultaneously, is normally present by what age? a. 6 to 8 months b. 12 months c. 1 month d. 3 to 4 months

d. 3 to 4 months

Which condition is caused by a virus that primarily infects a specific subset of T lymphocytes, the CD4+ T-cells? Select one: a. Idiopathic thrombocytopenic purpura (ITP) b. Wiskott-Aldrich syndrome c. Severe combined immunodeficiency disease d. Acquired immunodeficiency syndrome (AIDS)

d. Acquired immunodeficiency syndrome (AIDS)

What is the most appropriate nursing diagnosis for a child diagnosed with moderate anemia? Select one: a. Risk for injury related to depressed sensorium b. Decreased cardiac output related to abnormal hemoglobin c. Risk for Injury related to dehydration and abnormal hemoglobin d. Activity intolerance related to generalized weakness

d. Activity intolerance related to generalized weakness

In which condition are all the formed elements of the blood simultaneously depressed? Select one: a. Thalassemia major b. Iron deficiency anemia c. Sickle cell anemia d. Aplastic anemia

d. Aplastic anemia

When should a child diagnosed with cognitive impairment be referred for stimulation and educational programs? Select one: a. At age 3 years, when schools are required to provide services. b. At age 5 or 6 years, when schools are required to provide services. c. As soon as they have the ability to communicate in some way. d. As young as possible.

d. As young as possible.

Which situation poses the greatest challenge to the nurse working with a child and family? a. Rehabilitation admission b. Twenty-four-hour observation c. Outpatient admission d. Emergency hospitalization

d. Emergency hospitalization

What is the priority nursing intervention for a child hospitalized with hemarthrosis resulting from hemophilia? Select one: a. Assessment of the impact of hospitalization on the family system b. Assessment of the child's response to hospitalization c. Administration of acetaminophen for pain relief d. Immobilization and elevation of the affected joint

d. Immobilization and elevation of the affected joint

What food choice by the parent of a 2-year-old child with celiac disease indicates a need for further teaching? a. Corn muffin b. Meat patty c. Rice cake d. Oatmeal

d. Oatmeal

Which of the following statements is true regarding factors that influence pain? a. Expectation about how painful an event will be does not affect how much pain is felt b. Anxiety affects the level of pain only if the anxiety is about the event causing the pain c. Using restraints can help decrease pain during a procedure d. Use of a magic blanket is a pain control strategy the would assist an 8-year-old

d. Use of a magic blanket is a pain control strategy the would assist an 8-year-old

Which action is most likely to encourage parents to talk about their feelings related to their child's illness? a. Use direct questions. b. Be sympathetic. c. Avoid periods of silence. d. Use open-ended questions.

d. Use open-ended questions.

What major complication is noted in a child with chronic renal failure? Select one: a. Metabolic alkalosis b. Excessive excretion of blood urea nitrogen c. Hypokalemia d. Water and sodium retention

d. Water and sodium retention

The nurse educates the parents of a toddler-age client regarding play and toys appropriate for this stage of development. Which parental responses indicate correct understanding of the information presented? Select all that apply. 1 "We should expect our child to participate in parallel play." 2 "We should provide our child with finger paints to foster creativity." 3 "We should allow our child to watch as much television as she wants." 4 "We should provide our child with toys that foster her imagination, such as a doll." 5 "We should provide our child with toys so that we are able to finish household chores."

1 "We should expect our child to participate in parallel play." 2 "We should provide our child with finger paints to foster creativity." 4 "We should provide our child with toys that foster her imagination, such as a doll."

When assessing a toddler with Autism Spectrum Disorder (ASD), what characteristic findings or behaviors should the nurse expect? Select all that apply. 1 The desire to hug the nurse 2 Flat, blank facial expression 3 Laughing when pulse is taken 4 Inability to maintain eye contact 5 Enjoys climbing on stairs and furniture

2 Flat, blank facial expression 3 Laughing when pulse is taken 4 Inability to maintain eye contact

How can a nurse best soothe a hospitalized infant who appears to be in pain? 1 Feeding the infant 2 Holding the infant 3 Playing soft music in the room 4 Providing a quiet environment

2 Holding the infant

A 1-year-old infant is brought to the pediatric clinic for the first time. During the assessment the nurse suspects a developmental delay. What developmental milestone should have been achieved by this age? 1 Saying six words 2 Responding to peek-a-boo 3 Building a tower of two cubes 4 Pointing to things when they are named

2 Responding to peek-a-boo

A 2-month-old infant is admitted to the pediatric unit with a diagnosis of respiratory syncytial virus infection. The nurse plans to position the infant to improve the respiratory effort. What positions are best? Select all that apply. 1 Prone 2 Semi-Fowler 3 Trendelenburg 4 Hyper-extended head 5 Head in sniffing position

2 Semi-Fowler 5 Head in sniffing position

A 9-year-old child is found to have acute glomerulonephritis after a recent infection. What microorganism should the nurse suspect as the cause of the child's current health problem? 1 Haemophilus 2 Streptococcus 3 Pseudomonas 4 Staphylococcus

2 Streptococcus

What should the nurse include in a teaching plan for the parents of a child with vesicoureteral reflux? a. The need for the child to hold urine for 6 to 8 hours b. The use of bubble baths as an incentive to increase bath time c. Suggestions for how to maintain fluid restrictions d. The importance of taking prophylactic antibiotics

d. The importance of taking prophylactic antibiotics

The nurse understands that hypospadias refers to what urinary anomaly? a. Penis shorter than usual for age. b. Urethral opening along dorsal surface of penis. c. Absence of a urethral opening. d. Urethral opening along ventral surface of penis.

d. Urethral opening along ventral surface of penis.

A parent tearfully tells a nurse, "They think our toddler is developmentally delayed. We're investigating a preschool program for cognitively impaired children." What is the most appropriate response by the nurse? 1 Praising the parent for the decision and encouraging the plan 2 Asking for more specific information related to the developmental delays 3 Advising the parent to have the healthcare provider help choose an appropriate program 4 Explaining that this action may be premature and that the developmental delays could disappear

2 Asking for more specific information related to the developmental delays More information is needed. The term developmental delay suggests that some milestones for age are not being met at the average time; it is not synonymous with cognitive impairment.

A nurse is caring for a 2½-year-old child who is expressing pain. What is the most reliable indicator of this child's pain? 1 Crying and sobbing 2 Changes in behavior 3 Verbal exclamations of pain 4 Changes in pulse and respiratory rate

2 Changes in behavior Although there are several indicators of pain in children, a change in behavior is the one that occurs most often. Crying is not a valid indicator of pain; there is more than one cause for crying, including pain, separation, fear, and unhappiness.

A 3-month-old infant has been hospitalized with respiratory syncytial virus (RSV). What is the priority intervention? 1 Administering an antiviral agent 2 Clustering care to conserve energy 3 Offering oral fluids to promote hydration 4 Providing an antitussive agent whenever necessary

2 Clustering care to conserve energy Often the infant will have a decreased pulmonary reserve, and the clustering of care is essential to provide for periods of rest.

A 2-year-old boy born with cryptorchidism is to undergo orchiopexy. What should the nurse tell the parents about the anticipated outcome of this surgery? 1 The urine stream will be directed downward. 2 Damage to the undescended testicle will be prevented. 3 Fluid that has collected in the scrotum will be removed. 4 The fibrous tissue that has caused the penile deformity will be released.

2 Damage to the undescended testicle will be prevented.

On the third day of a 2-year-old toddler's hospitalization the nurse notes that the child, who had been screaming and crying inconsolably, has begun to regress and is now lying quietly in the crib with a blanket. What stage of separation anxiety has developed? 1 Denial 2 Despair 3 Mistrust 4 Rejection

2 Despair The second stage of separation anxiety is despair, in which the child is depressed, lonely, and uninterested in the surroundings.

The school nurse is working with a child with a hearing deficit. The child arrives at school today without hearing aids. When the nurse talks with the child about the reasons for not wearing the aids, the nurse will need to ensure that the child understands what is being said. What actions by the nurse will promote effective communication? Select all that apply. 1 Speaking slower, louder than normal, and excessively fast 2 Facing the child directly when talking to the child 3 Avoiding chewing gum while communicating with the child 4 Avoiding using hand expressions that could interfere with lip reading 5 Moving from side to side while talking to the child to keep the child looking at the nurse

2 Facing the child directly when talking to the child 3 Avoiding chewing gum while communicating with the child

An infant is admitted to the pediatric unit with bronchiolitis caused by respiratory syncytial virus (RSV). What interventions are appropriate nursing care for the infant? Select all that apply. 1 Limiting fluid intake 2 Instilling saline nose drops 3 Maintaining contact precautions 4 Suctioning mucus with a bulb syringe 5 Administering warm humidified oxygen

2 Instilling saline nose drops 3 Maintaining contact precautions 4 Suctioning mucus with a bulb syringe Saline nose drops help clear the nasal passage, which improves breathing and aids the intake of fluids. RSV is contagious; infants with RSV should be isolated from other children, and the number of people visiting or caring for the infant should be limited. Infants with RSV produce copious amounts of mucus, which hinders breathing and feeding; suctioning before meals and at naptime and bedtime provides relief.

A home health nurse is caring for school-aged children in a family that is economically deprived. Which characteristic is most common to those living in poverty? 1 Open expression of anger 2 Long-term feeling of powerlessness 3 Willingness to postpone gratification 4 Compliance with health recommendations

2 Long-term feeling of powerlessness

A nurse is caring for a 6-year-old child who is admitted to the pediatric unit with recently diagnosed nephrotic syndrome. The parents ask the nurse why their child is retaining so much fluid. What should the nurse consider before telling the parents about the changes in body fluid distribution in language that they can understand? 1 Loss of sodium and water through an impaired basement membrane of the glomerulus results in hypovolemia. 2 Loss of body protein reduces oncotic pressure, and fluid moves from the intravascular to the interstitial space. 3 Hyperproteinemia results in increased oncotic pressure, and fluid moves from the intravascular to interstitial space. 4 Basement membranes of the glomeruli become selectively impermeable to water, and fluid is retained in the tissues.

2 Loss of body protein reduces oncotic pressure, and fluid moves from the intravascular to the interstitial space.

A 6-month-old infant is brought to the emergency department in severe respiratory distress. A diagnosis of respiratory syncytial virus (RSV) infection is made, and the infant is admitted to the pediatric unit. What should be included in the nursing plan of care? 1 Place in a warm, dry environment. 2 Maintain standard and contact precautions. 3 Administer prescribed antibiotic immediately. 4 Allow parents and siblings to room in with the infant.

2 Maintain standard and contact precautions.

A 2-year-old toddler has hearing loss caused by recurrent otitis media. What treatment does the nurse anticipate that the practitioner will recommend? 1 Ear drops 2 Myringotomy 3 Mastoidectomy 4 Steroid therapy

2 Myringotomy Myringotomy is a surgical opening into the eardrum to permit drainage of accumulated fluid associated with otitis media.

Two 3-year-old clients are playing together in a hospital playroom. One is working on a puzzle, and the other is stacking blocks. Which type of play are these children participating in based on this scenario? 1 Solitary play 2 Parallel play 3 Associative play 4 Cooperative play

2 Parallel play

A nurse is discussing the care of an infant with colic. What should the nurse explain to the parents is the cause of colicky behavior? 1 Inadequate peristalsis 2 Paroxysmal abdominal pain 3 An allergic response to certain proteins in milk 4 A protective mechanism designed to eliminate foreign proteins

2 Paroxysmal abdominal pain The traditional efforts to explain and treat colic center on the paroxysmal abdominal pain; multiple factors appear to be involved, including immaturity of the intestinal nervous system and lack of normal intestinal flora.

A 15-year-old with cystic fibrosis (CF) is admitted with a respiratory infection. The nurse determines that the adolescent is cyanotic, has a barrel-shaped chest, and is in the 10th percentile for both height and weight. What is the priority nursing intervention? 1 Increasing physical activities 2 Performing postural drainage 3 Maintaining dietary restrictions 4 Administering prescribed pancreatic enzymes

2 Performing postural drainage Postural drainage, including percussion and vibration, aids removal of respiratory secretions that provide a medium for further bacterial growth.

A nurse is assessing the condition of a school-aged child with acute glomerulonephritis. What clinical finding does the nurse anticipate? 1 Ketonuria 2 Periorbital edema 3 Increased appetite 4 Decreased blood pressure

2 Periorbital edema

The nurse is planning care for a school-aged child with autism spectrum disorder (ASD) who has been hospitalized for some tests. Which intervention should the nurse plan to implement? 1 Providing adequate stimulation through play 2 Placing the child in a private room 3 Encouraging staff to visit the child frequently 4 Giving detailed explanations about the upcoming tests

2 Placing the child in a private room The child with ASD should be placed in a private room. Decreasing stimulation by placing the child in a private room may lessen the disruptiveness of hospitalization. Play should be carefully planned; overstimulation can precipitate behavioral outbursts. Children with ASD need to be introduced slowly to new situations, with visits from staff caregivers kept short whenever possible.

A toddler who was admitted in acute respiratory distress is now resting quietly. The parents tell the nurse that they must leave. What should the nurse suggest that the parents do? 1 Try to "room in" to decrease the child's anxiety. 2 Plan to visit the child as frequently as possible. 3 Tell the child that they are leaving but will be back tomorrow. 4 Leave while the child is distracted to reduce the upset it may cause.

2 Plan to visit the child as frequently as possible. Because the parents have stated that they must leave, advising them to visit as often as possible takes into consideration the effect that separation will have on the toddler.

A school-aged child with cystic fibrosis has been admitted with a respiratory infection. The child has been very disruptive and angry with staff and parents. What does the nurse suspect is the reason for the child's uncooperative behavior? 1 Spoiled and needs to be adequately disciplined 2 Resentful of the restriction of the hospitalizations 3 Having a reaction to the new respiratory medications 4 Angry about dietary restrictions related to the disease

2 Resentful of the restriction of the hospitalizations Children with cystic fibrosis often become resentful of repeated hospitalizations, the disease itself, and restrictions on their activities.

Which data collected during the nursing assessment for a 24-month-old client indicates the need for further evaluation for delayed language? Select all that apply. 1 The child uses two-word sentences. 2 The child cries and points at an object he wants. 3 The child states, "Me do it" when asked to stack blocks. 4 The child understands the meaning of as many as 50 words. 5 The child asks, "What's that?" when the nurse uses the stethoscope to assess lung sounds.

2 The child cries and points at an object he wants 4 The child understands the meaning of as many as 50 words. Observations made during the nursing assessment for a 24-month-old client that would require further evaluation for a language delay include: the child crying and pointing at an object that is wanted and the child who understands the meaning of only 50 words (300 is expected by this stage of development). The use of two-word sentences, "Me do it," and asking "What's that?" are all expected findings for the 24-month-old client in regards to language development.

What strategy should the nurse employ to be effective when using play therapy with a 6-year-old child with autism? 1 Play music and dance with the child. 2 Use mechanical and inanimate objects for play. 3 Employing positive reinforcements such as hugging. 4 Provide brightly colored toys and blocks that can be held.

2 Use mechanical and inanimate objects for play. Self-isolation and disinterest in interpersonal relationships lead the autistic child to find security in nonthreatening, impersonal objects.

During assessment, the nurse asks a client about developmental milestones such as the age at which thelarche and menarche occurred. The nurse determines that the client experienced pubertal delay. Which finding in the client's history supports the nurse's conclusion? 1 Weight increased by 8 to 12 kg. 2 Menarche occurred 2 years after thelarche. 3 Breast development occurred by 15 years of age. 4 Growth in height stopped 2 years after menarche.

3 Breast development occurred by 15 years of age. When the development of breasts has not occurred by 13 years of age in girls, it is considered pubertal delay.

A nurse is caring for an infant with Down syndrome. What does the nurse recall as the most common serious anomaly associated with this disorder? 1 Renal disease 2 Hepatic defects 3 Congenital heart disease 4 Endocrine gland malfunction

3 Congenital heart disease

An emergency tracheotomy is performed on a toddler in acute respiratory distress from laryngotracheobronchitis (viral croup). What early signs of respiratory distress indicate that it is necessary for the nurse to suction the tracheotomy? Select all that apply. 1 Stridor 2 Cyanosis 3 Restlessness 4 Increased pulse rate 5 Substernal retractions

3 Restlessness 4 Increased pulse rate Restlessness and increased pulse rate are early signs of hypoxia; suctioning is required to keep the airway patent. Stridor, cyanosis, and substernal retractions are late signs of hypoxia; suctioning should be performed before substernal retractions occur.

A newborn has just been admitted to the pediatric surgical unit from the birth hospital with a diagnosis of tracheoesophageal fistula. In what position should this child be maintained? 1 Prone, to reduce risk of aspiration 2 Trendelenburg, to drain stomach contents 3 Semi-Fowler, to reduce the risk of chemical pneumonia 4 Supine, to reduce the risk of sudden infant death syndrome

3 Semi-Fowler, to reduce the risk of chemical pneumonia

Which should the nurse anticipate for a 5-year-old client related to play? Select all that apply. 1 Solitary play 2 Parallel play 3 Dress-up play 4 Team sport play 5 Cooperative play

3 Dress-up play 5 Cooperative play The 5-year-old preschool-age client would be expected to participate in dress-up play due to the preoccupation with magical thinking and the beginnings of cooperative play. Solitary play is expected for the infant and toddler. Parallel play is expected for the toddler and young preschool-age client. Team sport play is expected for the school-age client.

After abdominal surgery, a 5-year-old child is experiencing pain, and an opioid analgesic is prescribed. What should the nurse consider about children in pain and their response to opioid analgesics when an opioid analgesic is prescribed? 1 Addiction to opioids is more of a risk for children than adults. 2 Analgesics are not needed as frequently because pain is not as strongly felt by children as it is by adults. 3 Even though children do not like taking medicines, analgesics will make them more comfortable. 4 Children do not need analgesics because they are easily distracted and will quickly return to play or sleep.

3 Even though children do not like taking medicines, analgesics will make them more comfortable.

A nurse is planning to teach the parents of a preschool child with recently diagnosed cystic fibrosis why the child has respiratory problems. What should the nurse remember about the underlying pathophysiology? 1 Airway irritability causes spasms. 2 Lung parenchyma becomes inflamed. 3 Excessively thick mucus obstructs airways. 4 Endocrine glands secrete surplus hormones.

3 Excessively thick mucus obstructs airways.

The mother of an infant with Down syndrome asks the nurse what causes the disorder. Before responding, the nurse recalls that the genetic factor of Down syndrome results from what? 1 An intrauterine infection 2 An X-linked genetic disorder 3 Extra chromosomal material 4 An autosomal recessive gene

3 Extra chromosomal material

A nurse is caring for a school-aged child with nephrotic syndrome who has massive edema. The nurse teaches the parents about the low-sodium diet that has been ordered. Which food group has the lowest level of sodium compared with the other food groups? 1 Meat 2 Dairy 3 Fresh fruit 4 Fresh vegetables

3 Fresh fruit

A 13-month-old toddler has a respiratory tract infection with a low-grade fever. When teaching the parents, which intervention should the nurse emphasize? 1 Encouraging high-calorie snacks to prevent weight loss 2 Keeping the toddler wrapped in blankets to prevent shivering 3 Giving small amounts of clear liquids frequently to prevent dehydration 4 Using cool-water baths to prevent the toddler's fever from increasing further

3 Giving small amounts of clear liquids frequently to prevent dehydration

A 9-year-old child is admitted to the pediatric unit with a diagnosis of acute glomerulonephritis. What does the nurse expect the admission urinalysis to reveal? 1 Polyuria 2 Ketonuria 3 Hematuria 4 Bacteriuria

3 Hematuria The urine is cloudy, smoky, or the color of tea because of the presence of erythrocytes and casts from the affected kidney tissue.

A nurse anticipates that dialysis will be necessary for a 12-year-old child with chronic kidney disease when the child begins to exhibit which symptom? 1 Hypotension 2 Hypokalemia 3 Hypervolemia 4 Hypercalcemia

3 Hypervolemia Hypervolemia results when the kidneys have failed and are no longer able to maintain homeostasis, the blood pressure is high, and cardiac overload is imminent.

During the assessment of a hospitalized infant, the nurse notes dry mucous membranes, absence of tears when the infant cries, and poor skin turgor. Which parameter will help the nurse further evaluate these findings? 1 Daily serum electrolytes 2 Respiratory rate and rhythm 3 Intake and output over the past 24 hours 4 Alterations in heart sounds since admission

3 Intake and output over the past 24 hours

A nurse is teaching a class for staff members working in a group home about the cognitive development of children with cognitive impairments. What concept can these children probably learn the fastest? 1 Love versus hate 2 Life versus death 3 Large versus small 4 Right versus wrong

3 Large versus small

The mother of a 2-year-old child tells the nurse that she is concerned about her child's vision. What behavior when the child is tired leads the nurse to suspect strabismus? 1 One eyelid droops. 2 Both eyes look cloudy. 3 One eye moves inward. 4 Both eyes blink excessively.

3 One eye moves inward.

A 6-year-old child is admitted to the pediatric unit with a diagnosis of nephrotic syndrome. What should the plan of care include during the acute phase? 1 Offering a low-protein diet 2 Encouraging fluids every hour 3 Promoting frequent position changes 4 Providing time for active play periods

3 Promoting frequent position changes Severe edema is usually present, and changes of position are necessary to prevent skin breakdown. A high-protein diet should be offered, although there is no evidence that it alters the outcome of the disorder. A low-protein diet is used for children with azotemia resulting from renal failure.

Elbow restraints are prescribed for an 18-month-old toddler who just had surgery for a cleft palate. The nurse explains to the parents that the restraints are used to keep the child from doing what? 1 Playing with unsterile toys 2 Rolling to a supine position 3 Putting fingers into the mouth 4 Removing the nasogastric tube

3 Putting fingers into the mouth

A 1-year-old infant with a distended abdomen is admitted to the pediatric unit with the diagnosis of Hirschsprung disease. In which position should the nurse place the infant? 1 Prone 2 Sitting 3 Supine 4 Lateral

4 Lateral In the lateral position the distended abdomen does not press against the diaphragm, facilitating lung expansion.

Which neurologic manifestation should the nurse anticipate for a toddler-age client exposed to low doses of lead? 1 Coma 2 Paralysis 3 Convulsions 4 Learning difficulties

4 Learning difficulties

A nurse is caring for a 6-year-old child with a diagnosis of glomerulonephritis. The child's urine output decreases to less than 100 mL/24 hr, the creatinine clearance is 60 mL/min, and there is an irregular apical pulse. A diagnosis of acute renal failure is made. Blood is drawn for testing. Which serum level requires immediate intervention? 1 Sodium 126 mEq/L (126 mmol/L) 2 Bilirubin 0.3 mg/dL (5.1 mcmol/L) 3 Creatinine 1.3 mg/dL (114.4 mcmol/L) 4 Potassium 6.1 mEq/L (6.1 mmol/L)

4 Potassium 6.1 mEq/L (6.1 mmol/L)

The nurse is providing education to the parents of a preschool-age client who is experiencing a severe fear of the dark. Which treatment option should the nurse share with the parents during the teaching session? 1 Prescription medication 2 Electroconvulsive therapy 3 Intensive therapy sessions 4 Repetition of brave statements

4 Repetition of brave statements

A 15-year-old adolescent with Down syndrome is scheduled for surgery. The parents inform the nurse that their child has a mental age of 8 years. At what age level should the nurse prepare the child's preoperative teaching plan? 1 Adult, for the parents to understand 2 Specific age, as ordered by the healthcare provider 3 Adolescent, because this is the child's chronologic age 4 School-age, because this is the child's developmental age

4 School-age, because this is the child's developmental age

The parents of a boy with hypospadias with chordee ask a nurse why their child should undergo corrective surgery. What problem that may develop eventually should the nurse discuss with the parents? 1 Renal failure 2 Testicular cancer 3 Testicular torsion 4 Sexual difficulties

4 Sexual difficulties

During a well-child visit the parents tell a nurse, "Our 3-year-old doesn't listen to us when we speak and ignores us!" An auditory screening reveals that the child has a mild hearing loss. What should the nurse explain to the parents about this degree of hearing loss? 1 A severe hearing deficit may develop. 2 It will not interfere with progress in school. 3 An immediate follow-up visit is not necessary. 4 Speech therapy in addition to hearing aids may be required.

4 Speech therapy in addition to hearing aids may be required.

A mother brings her 6-year-old child to the pediatric clinic, stating that the child has not been feeling well, is weak and lethargic, and has a poor appetite, headaches, and smoky-colored urine. What additional information should the nurse obtain that will aid diagnosis? 1 Rash on palms and feet 2 Shoulder and knee pain 3 Recent weight loss of 2 lb (0.9 kg) 4 Strep throat in the past two weeks

4 Strep throat in the past two weeks The smoky urine and the stated symptoms should lead the nurse to suspect glomerulonephritis, which usually occurs after a recent streptococcal infection.

A nurse is caring for an infant who has undergone surgery to repair a diaphragmatic hernia. What is the best position for the nurse to place the infant in? 1 Semi-Fowler in an infant seat 2 Side-lying on the unaffected side 3 Prone with the head turned to the side 4 Supine with the head of the bed elevated

4 Supine with the head of the bed elevated The supine position keeps pressure off the surgical site. Placing the infant on the unaffected side limits gas exchange in the lung on the unoperated side.

The nurse is providing discharge instructions to the parents of a child who has undergone surgical correction of hypospadias. What is the priority information for the nurse to include? 1 Ensuring that the child's privacy is maintained 2 Increasing the time that the catheter is clamped 3 Maintaining the surgically implanted tension device 4 Teaching parents how to care for the catheterization system

4 Teaching parents how to care for the catheterization system Parents should know how to empty the urine bag and how to prevent kinking of the tubing.

A toddler undergoes the implantation of a low-profile (skin-level) device (button) for a gastrostomy. The gastrostomy is now healed, and the parents are being taught to care for the stoma. What parental behavior indicates to the nurse that additional teaching is needed? 1 A parent is cleaning the stoma with soapy water. 2 Gastric contents are aspirated before the start of a feeding. 3 A parent inserts an adapter into the button to initiate a feeding. 4 The button is being maintained in the same position within the stoma.

4 The button is being maintained in the same position within the stoma.

A nurse receives an order to give amoxicillin/clavulanate 50 mg/kg q4h to a 4-year-old child who weighs 35 pounds (16 kg). The medication label says that there is 200 mg of amoxicillin/clavulanate in 1 mL of normal saline. How much amoxicillin/clavulanate should the nurse give? Record your answer using a whole number.

4 mL

Children's patterns of play change as they grow from infancy through school age. Rank the order of appearance of each type of play, starting with infant play. 1. Associative 2. Cooperative 3. Parallel 4. Solitary

4, 3, 1, 2

What is the difference between asthma rescue/relief medications and controller medications?

Relief- is short acting, taken during exacerbation Control- is long acting, treats underlying things- maybe controls allergy response or decreases inflammation in general at a really low dose, effects mucous production, taken regardless of symptoms

An infant's parents ask the nurse about preventing otitis media (OM). What intervention should the nurse recommend? Select one: a. Avoid tobacco smoke b. Bottle-feed or breastfeed in supine position c. Avoid children with OM d. Use nasal decongestant

a. Avoid tobacco smoke

Which diagnostic finding is present when a child has primary nephrotic syndrome? Select one: a. Positive ASO titer b. Leukocytosis c. Proteinuria d. Hyperalbuminemia

c. Proteinuria

How is LMX or Emla used in pediatric pain management? Select one or more: a. It should be injected subcutaneously near the preferred injection site prior to the painful procedure. b. It should be rubbed into the skin prior to the painful procedure. c. It should be applied to skin in a thick layer and covered with an occlusive dressing before the painful procedure. d. It should be given on an empty stomach before a painful procedure.

i got this wrong lol


Kaugnay na mga set ng pag-aaral

ISACA Studying CyberSecurity Fundamentals

View Set

Dr mcallens classs american indian tribes utah studies TEST :( :( :(

View Set

Chap 15 HR Compensation & Benefits

View Set

Chapter 40: Musculoskeletal Care Modalities

View Set

Chapter 2: Basic Concepts and Processes PrepU

View Set

Lippincott the child with cardio health problems

View Set